pallimed/geripal blogs to boards - hospice/palliative medicine board review 2012 (q&a +...

48
From Blogs to Boards: AAHPM 2012 PreCourse from the contributors of Pallimed & Geripal Page 1 of 48 From Blogs to Boards: Answer Key Editors: Suzana Makowski Drew Rosielle Paul Tatum Eric Widera Christian Sinclair

Upload: christian-sinclair-md

Post on 02-Nov-2014

487 views

Category:

Health & Medicine


1 download

DESCRIPTION

Blogs to Boards Created by Pallimed and GeriPal contributors in 2012 as a free study tool for the 2012 Hospice and Palliative Medicine board certification test. Creative Commons license - you must include attribution and links to Pallimed and GeriPal, and cannot reproduce for any commercial use. We have posted the questions without the answers separately if you are looking for those.

TRANSCRIPT

Page 1: Pallimed/GeriPal Blogs to Boards - Hospice/Palliative Medicine Board Review 2012 (Q&A + Discussion)

From  Blogs  to  Boards:  AAHPM  2012  Pre-­‐Course  from  the  contributors  of  Pallimed  &  Geripal  

Page 1 of 48  

From Blogs to Boards: Answer Key Editors:

• Suzana Makowski • Drew Rosielle • Paul Tatum • Eric Widera • Christian Sinclair

Page 2: Pallimed/GeriPal Blogs to Boards - Hospice/Palliative Medicine Board Review 2012 (Q&A + Discussion)

From  Blogs  to  Boards:  AAHPM  2012  Pre-­‐Course  from  the  contributors  of  Pallimed  &  Geripal  

Page 2 of 48  

From Blogs to Boards Goals of this session:

§ Prepare for the boards with an audience response quizz hosted by Pallimed & Geripal bloggers § Review recent, clinically relevant literature on Palliative Care § Discover three methods to obtain quality up-to-date information on palliative care

Board Review Content:

Board Content

8%7%

6%

45%

5%

9%

11%

9%Approach to CarePsychosocial & Spiritual ConcernsImpending DeathGrief & BereavementMedical ManagementCommunication & TeamworkEthical & Legal Decision-MakingPrognostication

Page 3: Pallimed/GeriPal Blogs to Boards - Hospice/Palliative Medicine Board Review 2012 (Q&A + Discussion)

From  Blogs  to  Boards:  AAHPM  2012  Pre-­‐Course  from  the  contributors  of  Pallimed  &  Geripal  

Page 3 of 48  

Notes Question HPM1

Ms. V is a 68 year old with metastatic non-small cell lung cancer, congestive heart failure, and mild renal insufficiency residing in an inpatient palliative care unit for management of bone pain. Her medications include morphine IR, fentanyl transdermal patch, furosemide, senna, and Fleet enema’s prn. Ms. V did not have a bowel movement in 4 days. Basic labs were ordered for the next morning as well as a two of her prn enemas, although they failed to result in a bowel movement. The labs the next day reveal a serum sodium of 124, potassium of 3.0, creatinine of 1.4 (baseline of 1), low calcium of 6.5, and a very elevated phosphate of 17 mg/dl. What is the most likely cause of her electrolyte abnormalities?

a) A medication adverse event b) Tumor lysis syndrome c) Bowel Impaction d) Osteolytic metastases

Discussion: Correct answer is (a)

a) Sodium phosphate preparations should never be given to patients with renal insufficiency, heart failure, cirrhosis, or elderly frail individuals due to significant risks of adverse effect. Both oral and rectal sodium phosphate preparations can cause significant fluid shifts within the colon resulting in intravascular volume depletion. Furthermore, these preparations can cause electrolyte disturbances including significant hyperphosphatemia, hypocalcemia, and hypokalemia. A significant clinically important rise in serum phosphate can even be seen in elderly patients with normal renal function. (J Gastroenterol Hepatol. 2004;19(1):68). Lastly, phosphate nephropathy may occur due to the transient and potentially severe increase in serum phosphate combined with volume depletion from the fluid shifts.

b) Tumor lysis may indeed cause hyperphosphatemia and hypocalcemia, although it is generally seen in with cytotoxic therapy in patients with a large tumor burden with rapid cell turnover (ie. Non-Hodgkins Lymphoma or certain leukemias). It is also associated with hyperkalemia.

c) Bowel impaction alone should not cause these electrolyte disturbances d) Osteolytic metastases generally cause hypercalcemia.

References:

• http://www.geripal.org/2012/02/dangers-of-fleet-enemas.html

Page 4: Pallimed/GeriPal Blogs to Boards - Hospice/Palliative Medicine Board Review 2012 (Q&A + Discussion)

From  Blogs  to  Boards:  AAHPM  2012  Pre-­‐Course  from  the  contributors  of  Pallimed  &  Geripal  

Page 4 of 48  

Notes Question HPM2 Walking into a room at your hospice inpatient unit you see a tired appearing female patient

lying in bed with soft moaning, holding her abdomen. She has end stage CHF and no history of cancer. Review of your notes show decreasing oral intake and increased time in bed. Her nurse reports she disimpacted her yesterday after suppositories and enemas were ineffective for worsening constipation. Medications include: Fentanyl 50mcg patch (on for several weeks), Senna 2 tabs BID, Colace daily, Recent enema, and docusate suppository Exam: Cachectic female, Scaphoid abdomen, hypoactive bowel sounds, formed (but not hard) stool on rectal exam. What is the next best step?

a) Write an order for methylnaltrexone 8mg subcutaneously x1 now. b) Switch her from a fentanyl patch to a morphine pump so you can better manage her

abdominal pain. c) Write an order for octreotide 200mcg subcutaneously twice daily for three days d) Place an NG and give her polyethylene glycol daily until she has a bowel movement or

regains ability to swallow and you can remove the NG tube Discussion: Correct answer is (a)

a) The patient likely has opioid induced constipation (OIC). Methylnaltrexone is a mu-opioid receptor antagonist and is related to naloxone. After ruling out bowel obstruction, fecal impaction and any other abdominal process, you give methylnaltrexone at 0.15mg/kg subcutaneously, usually 8 (patients < 136lbs) or 12 mg (patients over 136lbs). About 60 percent of patients will have a BM in under 4 hours. Usually within 30 minutes of the first dose. Number needed to treat was 2.2 (pretty darn good). One barrier is cost. At $48 per 8mg dose this is a costly way to manage constipation.

b) While controlling abdominal pain is important relieving the cause of the abdominal pain takes precedence. Opioids may be the cause of her pain – increasing them is not indicated. With the exception of imminently dying patients, proper treatment of OIC will lead to its resolution and function can be improved.

c) Octreotide has a role in palliative care for malignant bowel obstruction (MBO), not constipation. This patient does not have a cancer history and sudden onset nausea and vomiting that may be signs for a MBO. Octreotide also is expensive-costing between $40 and $80 per dose.

d) Placing a nasogastric tube should be avoided whenever possible when there are less invasive measures available. The patient can swallow oral laxatives, and does not have an MBO and so does not have any minimal indications for an NGT in any case. Polyethylene glycol is helpful as an osmotic laxative and is often employed as a first line option for OIC. It is often more helpful as part of a maintenance regimen or for mild to moderate constipation.

References:

• Thomas, Jay et. al. Methylnaltrexone for Opioid Induced Constipation in Advanced Illness. 2008. NEJM 358 (22): 2332-2343.

• Yuan, Chun-Su. Methylnaltrexone Mechanisms of Action and Effects on Opioid Bowel Dysfuction and Other Opioid Adverse Side Effects. The Annals of Pharmacotherapy, 2007. 41: 984- 993

Page 5: Pallimed/GeriPal Blogs to Boards - Hospice/Palliative Medicine Board Review 2012 (Q&A + Discussion)

From  Blogs  to  Boards:  AAHPM  2012  Pre-­‐Course  from  the  contributors  of  Pallimed  &  Geripal  

Page 5 of 48  

Notes Question HPM3

During a hospice interdisciplinary team meeting, you hear about a 53 year old resident of the local nursing home. He has ALS with bulbar attributes, and is starting to have difficulty swallowing and speaking. He is bedbound most of the day. He has had two episodes of aspiration pneumonia in the last month. His nurse describes the scene with the patient’s wife, Sally, at his side, squeezing his hand with one hand and her rosary with the other. He explained to the nurse, “I told Sally that I don’t want a feeding tube. I’ve had a good life and have few regrets. I saw my father-in-law die on a feeding tube and I would not want to go through that, or put my wife through that. But I am Catholic. Our friend at the parish said that I have to ‘do everything’ to prolong my life – especially when it comes to nutrition - or I will go hell. I don’t want to go to hell.” His wife nods emphatically. During the interdisciplinary care meeting, the chaplain (in his role as teacher) asks you to explain to the team what your understanding of the Catholic doctrine is as pertaining to this patient. What do you say?

a) My understanding is that medically assisted nutrition is obligatory for patients who are unable to take food by mouth.

b) My understanding is that medically assisted nutrition is morally optional for most patients at the end of life.

Discussion: Correct answer is (b)

a) Some interpret the teachings of the church to mandate artificial nutrition at the end of life, especially with the media coverage of Terri Schiavo. But the doctrine is more nuanced than that. “58. In principle, there is an obligation to provide patients with food and water, including medically assisted nutrition and hydration for those who cannot take food orally. This obligation extends to patients in chronic and presumably irreversible conditions (e.g., the “persistent vegetative state”) who can reasonably be expected to live indefinitely if given such care.” (from: section 58. Ethical and Religious Directives for Catholic Health Care Services.)

b) The discussion is actually more complex then that: Medically-assisted nutrition and hydration become morally optional when they cannot reasonably be expected to prolong life or when they would be “excessively burdensome for the patient or [would] cause significant physical discomfort, for example resulting from complications in the use of the means employed. “59. The free and informed judgment made by a competent adult patient concerning the use or withdrawal of life-sustaining procedures should always be respected and normally complied with, unless it is contrary to Catholic moral teaching.” (from: section 59. Ethical and Religious Directives for Catholic Health Care Services.)

References: http://www.pallimed.org/2010/01/catholic-directives-on-artificial.html http://www.pallimed.org/2008/08/media-coverage-of-terri-schiavo.html

Page 6: Pallimed/GeriPal Blogs to Boards - Hospice/Palliative Medicine Board Review 2012 (Q&A + Discussion)

From  Blogs  to  Boards:  AAHPM  2012  Pre-­‐Course  from  the  contributors  of  Pallimed  &  Geripal  

Page 6 of 48  

Notes Question HPM4 Mrs Dole, a 68 year old with 20 year history of Diabetes Mellitus Type II is referred to

Palliative Care from Oncology with Stage III Nasopharyngeal carcinoma. Nausea is the key concern. For last 3 years she has had early satiety but maintained weight. Since initiating chemotherapy, she has had nausea for the first 2 days of her chemotherapy cycle, which then resolves. 1 week after the last round of chemotherapy she required intravenous fluids for dehydration. Now 2 weeks later is having intermittent severe nausea. It can be provoked by sudden changes in body position. She fell once because she lost her balance. Usually she does not vomit, but occasionally does. She describes a feeling of the room spinning associated with the nausea. Of the following options, which drug is most targeted to this patient’s specific nausea type:

a) Ondansetron b) Prochlorperazine c) Metoclopramide d) Diazepam e) Meclizine

Discussion: (e) are the correct answers This patient has had multiple types of nausea, however currently her major nausea type seems to be vestibular. She may have developed an otolith while dehydrated. Some chemotherapeutic agents are ototoxic and can cause vestibular symptoms including hearing loss, tinnitus, vertigo/nausea. She also has had chemotherapy induced nausea, as well as diabetic gastroparsis. For the boards, probably the default choice for nausea will be D2 blockers, however there are certain types of nausea for which D2 blockers are not the best choice.

a) Ondansetron and the other ‘-setrons’ are HT3 receptor blockers and have excellent evidence for the treatment of chemotherapy induced nausea, and post-operative nausea. While used widely for other types of nausea including opioid-associated, there is less evidence to support them for these practices. They are exceedingly safe and well-tolerated; they are constipating. *** Chemotherapy induced nausea/vomiting is considered acute when it occurs <24h after chemo infusion, and delayed if >24h. Delayed n/v usually occurs in the several days after chemotherapy, but not weeks. First line treatments to prevent acute CINV including 5HT3 blockers and steroids. NK-1 blockers such as aprepitant and gluclocorticoids are also used, especially for mod-highly emetogenic chemo. NK-1 blockers and steroids also prevent delayed N/V; 5HT3 blockers less so. D2 blockers are no longer first line agents as 5HT3 blockers have clearly shown superior efficacy and safety. Doses of metoclopramide needed to be effective are 1-2mg/kg IV!

b) Prochlorperazine and other D2 blockers such as haloperidol target the Chemoreceptor trigger zone and D2 receptor. They are the work-horses of nausea treatment.

c) While the patient has some component of diabetic gastroparesis suggested by satiety and long history of DM, he is not bothered by emesis with meals. Metoclopramide targets D2 receptors primarily in the gut, and has some prokinetic features, but its role long-term for gastroparesis is controversial as it causes EPS such as tardive dyskinesia.

d) Diazepam and benzodiazepines are effective for anticipatory nausea/vomiting which occurs in ~25% of chemo patients. Behavorial/cognitive treatments, and integrative modalities are probably helpful too. Aggressive prevention of CINV can help prevent anticipatory n/v.

e) She has what seems to be vestibular symptoms. Anticholinergic drugs such as meclizine, scopolamine, promethazine, and even diphenhydramine are potential drugs. CNS side effects such as sedation, confusion; as well as orthostatis and xerostomia are worrisome side effects.

References: • http://www.pallimed.org/2007/09/vatican-tube-feeding-more-on-abigail.html • http://jama.ama-assn.org/content/298/10/1196.full.pdf+html • Hain TC, Uddin M. Pharmacological treatment of vertigo. CNS

Drugs. 2003;17:85–100. • http://www.oncolink.org/resources/article.cfm?c=16&s=59&ss=224&id=1004

Page 7: Pallimed/GeriPal Blogs to Boards - Hospice/Palliative Medicine Board Review 2012 (Q&A + Discussion)

From  Blogs  to  Boards:  AAHPM  2012  Pre-­‐Course  from  the  contributors  of  Pallimed  &  Geripal  

Page 7 of 48  

Notes Question HPM5 In hospice IDT, you discuss the case of a 68 year old female with ovarian cancer with abdominal

pain and sudden onset nausea and vomiting. She has had no recent bowel movements and is on minimal opioids. You suggest a trial of octreotide for a likely malignant bowel obstruction and the nurses say “Doctor! You say we can use octreotide for everything! Is there anything octreotide can’t be used for in hospice?” Which one of the following is not a potential scenario to use octreotide? Choose the best answer. Answers

a) A 37 year old male with end stage alcoholic hepatitis who starts vomiting blood b) A 90 year old with a severe diarrhea with a history of a rectal tumor and radiation burns to

the perineal area c) A 42 year old female with a tense distended abdomen leaking a small amount from a

previous paracentesis site. d) A 27 year old male with a malignant wound with copious drainage e) A 31 year old female with abdominal pain from opioid-induced constipation

Discussion: Answer is (e)

a) Octreotide is the Swiss Army Knife of palliative medications. It is a synthetic analog of somatostatin and has many mechanisms of action: in general, it has a global effect to decrease secretions primarily in the GI tract It can be costly as a medication alone but it could reduce the system cost by avoiding hospitalizations. You should talk with your local pharmacist to see about availability and cost in your local programs. It is typically administered via intermittent subcutaneous dosing. Study published in 2000 compared octreotide infusion with sclerotherapy and found that octreotide to be as effective as sclerotherapy regarding hemostasis at 48 hours and on day 7 after the index bleeding episode. So for the patient looking to avoid hospitalization with acute variceal bleed this may be a helpful (but expensive) medication.

b) While it does not work as a prophylactic treatment to prevent chemo and radiation induced diarrhea a few studies have shown that it can treat existing diarrhea related to these two common cancer treatments.

c) Rapidly accumulating ascites or situations where repeat paracentesis or drain may not be readily available have been shown to be responsive to octreotide. It also has been reported for use in pleural effusions related to cirrhosis.

d) Tumor related secretions have been show to respond to octreotide e) Indications for octreotide include (via palliativedrugs.com) : symptoms associated with

unresectable hormone-secreting tumors, e.g. carcinoid, VIPomas, glucagonomas and acromegaly; prevention of complications after elective pancreatic surgery; †bleeding esophageal varices; †salivary, pancreatic and enterocutaneous fistulas; †intractable diarrhea related to high output ileostomies, AIDS, radiotherapy, chemotherapy or bone marrow transplant;†inoperable bowel obstruction in patients with cancer; †hypertrophic pulmonary osteo-arthopathy;†ascites in cirrhosis and cancer; †buccal fistula; †death rattle (noisy respiratory secretions); †bronchorrhea;†reduction of tumor-related secretions.

References:

§ http://cases.pallimed.org/2008/06/am-i-really-going-to-have-to-live-like.html § http://www.pallimed.org/2008/11/octreotide-for-radiation-induced.html § Freitas DS, Sofia C, Pontes JM, Gregório C, Cabral JP, Andrade P, Rosa A, Camacho E,

Ferreira M, Portela F.... (2000) Octreotide in acute bleeding esophageal varices: a prospective randomized study. Hepato-gastroenterology, 47(35), 1310-4. PMID: 11100339

§ Kalambokis, G. (2006-01) Octreotide in the treatment of refractory ascites of cirrhosis. Scandinavian Journal of Gastroenterology, 14(1), 199-121. DOI: 10.1080/00365520510024043 M

§ Martenson et al. The efficacy of octreotide in the therapy of acute radiation-induced diarrhea: a randomized controlled study. International Journal of Radiation OncologyBiologyPhysics, 54(1), 195-202. DOI: 10.1016/S0360-3016(02)02870-5

Page 8: Pallimed/GeriPal Blogs to Boards - Hospice/Palliative Medicine Board Review 2012 (Q&A + Discussion)

From  Blogs  to  Boards:  AAHPM  2012  Pre-­‐Course  from  the  contributors  of  Pallimed  &  Geripal  

Page 8 of 48  

Notes Question HPM6 You visit a patient at home receiving hospice care for cancer. Her pain has been well controlled with

long acting morphine 60mg BID and occasional PRN doses of short acting liquid morphine (10mg) over the past few weeks: she had been tolerating this well. She has had recent progressive functional decline and is currently at a PPS of 20%. In the last 24 hours the patient has vomited and has been more lethargic and having difficulty swallowing pills. She appears uncomfortable. In your examination you see a very thin patient who appears to be dying with a prognosis in the few days to a week range. The patient’s son is a respiratory therapist at a hospital and is insisting you change the patient’s opioid to a fentanyl patch because “it is less sedating than morphine.” The best response is:

a) Because the patient is cachectic, you tell the family that fentanyl transdermal patches are not indicated because the medication will not be absorbed.

b) Agree with the son and convert the patient to a 37.5mcg/hr fentanyl patch with oral morphine liquid 10mg q1 hour PRN

c) Because the fentanyl will not be effective for over 24 hours, continue the long acting morphine sulfate 60mg BID but give it rectally instead of by mouth

d) Suggest starting a morphine infusion via her port at 1.7mg/hr basal with a 3mg q30min bolus PRN after talking with the son about his concerns about sedation.

Discussion: Answer is (d)

a) Cachexia has not been show to be a CLINICALLY RELEVALANT factor in absorption of transdermal fentanyl. Cachexia will decrease the amount of subcutaneous fat which is where fentanyl is stored AFTER absorption through the dermal layers. In 2009 Heiskanen did a study comparing blood levels between cachectic and non-cachectic volunteers and found no significant difference, although cachectic patients had a slightly lower mean concentration. There was no difference in VAS score.

b) Fentanyl is not less sedating than morphine at equianalgesic doses. Also there is no 37.5mcg/hr patch or 12.5mcg/hr patch. As written, and described by the manufacturer, the “12.5mcg/hr patch” is labeled and Rx’d as a “12mcg/hr” patch to prevent confusion with Rx’ing 125mcg/hr. As for the conversion, it could be acceptable to use a 25mcg/hr & 12mcg/hr patch (total 37mcg/hr) per the Fentanyl transdermal product insert. It recommends 25mcg/hr for someone on OMDD of 60-134mg and 50mcg/hr for someone on OMDD 135-224, so this is right in the middle. The Breitbart/Donner conversion of 2mg morphine = 1mcg/hr transdermal fentanyl which would be 60mcg/hr of fentanyl (You could choose 50 or 75 depending on other clinical circumstances).

c) The pharmacokinetics of fentanyl do not warrant switching to it if otherwise indicated. Morphine still has time to circulate and get out of her system, and fentanyl begins to reach significant blood concentrations 8-12 hours after application. If needed, she can be bridged with a few doses of liquid morphine. In addition, people do not prefer rectal administration if it could be avoided.

d) A morphine continuous infusion allows for the continuation of the current effective opioid in a patient who is likely not going to regain swallowing function. The conversion is most direct (120mg OMDD = 40mg daily IV = 1.7mg/hr (1.5 if your pumps are limited in decimal rates). A 3 mg IV morphine bolus most closely replicates the 10mg oral morphine doses that were effective prior. If you did not choose this answer because your hospice doesn’t use continuous infusions (expense, nurse familiarity, not available from local pharmacy) then start talking with your hospice to decrease these barriers to an effective and essential tool to good pain management.

References: • http://www.pallimed.org/2009/05/cachexia-and-absorption-of-transdermal.html • Heiskanen, Tarja. (2009-7) Transdermal fentanyl in cachectic cancer patients. PAIN, 70(1-2),

928-222. DOI: 10.1016/j.pain.2009.04.012 • Mercadante, Sebastiano. (2012-01-09) Sustained-release oral morphine versus transdermal

fentanyl and oral methadone in cancer pain management. European Journal of Pain, 7(Suppl.

Page 9: Pallimed/GeriPal Blogs to Boards - Hospice/Palliative Medicine Board Review 2012 (Q&A + Discussion)

From  Blogs  to  Boards:  AAHPM  2012  Pre-­‐Course  from  the  contributors  of  Pallimed  &  Geripal  

Page 9 of 48  

A), 320-1046. DOI: 10.1016/j.ejpain.2008.01.013 • Weissman DE. Converting to/from Transdermal Fentanyl, 2nd Edition. Fast Facts and

Concepts. July 2005; 2. Available at: http://www.eperc.mcw.edu/fastfact/ff_002.htm. • Tatum IV WO. (2002) Adult patient perceptions of emergency rectal medications for

refractory seizures. Epilepsy & behavior : E&B, 3(6), 535-538. PMID: 12609248 • Colbert SA, O'Hanlon D, McAnena O, & Flynn N. (1998) The attitudes of patients and health

care personnel to rectal drug administration following day case surgery. European journal of anaesthesiology, 15(4), 422-6. PMID: 9699099

• Mercadante, Sebastiano. (2012-01-09) Sustained-release oral morphine versus transdermal fentanyl and oral methadone in cancer pain management. European Journal of Pain, 7(Suppl. A), 320-1046. DOI: 10.1016/j.ejpain.2008.01.013

Page 10: Pallimed/GeriPal Blogs to Boards - Hospice/Palliative Medicine Board Review 2012 (Q&A + Discussion)

From  Blogs  to  Boards:  AAHPM  2012  Pre-­‐Course  from  the  contributors  of  Pallimed  &  Geripal  

Page 10 of 48  

Notes Question HPM7 JY, a 28 year old woman with advanced cystic fibrosis and Burkholderia cenocepacia

colonization is hospitalized for a cystic fibrosis exacerbation. She has chronic chest wall pain from coughing and pleurisy, and recently broke 2 ribs from coughing. She is on IV glucocorticoids, IV ketorolac, IV ketamine prior to vest treatments, and lorazepam. Prior to her hospitalization, she took oxycodone ER 30mg q12h. Currently she is on a hydromorphone IV PCA at 2mg/hour, with 2mg q30 minute boluses. She used 72mg of IV dilaudid in the last 24h. Despite this she is becoming drowsy, and reports her pain is minimally improved and still severe for most of the day: 7-8/10, and ‘nearly intolerable’ during vest therapy The best next step is to:

a) Increase her PCA basal and ‘bolus’ doses by 50% and monitor for 24 hours. b) Add a 5% lidocaine patch to her chest wall over her rib fractures c) Discontinue hydromorphone and switch the patient to another opioid d) Advise the primary team to stop vest therapies

Discussion: Correct answer (c)

a) Indications for opioid rotation are 1) dose-limiting side effects such as sedation, nausea, pruritus, myoclonus from the patient’s current opioid, 2) need for a new dosing route (patient cannot swallow), 3) costs/insurance changes, 4) inadequate analgesia despite ‘adequate’ dose-escalation of the current opioid. There is no consensus on what 4 actually means, however rapidly escalating someone by an order of magnitude (as in this case) without good response, is generally a scenario in which you’d consider rotation (if not long before). Is not best next step given the above discussion

b) No data at all suggesting the lidocaine patch is effective for pain from fractures c) Is the correct answer: Morphine, methadone, or fentanyl are all reasonable options.

Some prefer methadone in these sorts of settings, but no actual data to support that and probably not tested on the boards. Another reasonable approach in this situation would be to consult a pain interventionalist for regional options.

d) Opioid rotation is reasonable first, before advising this, as it will likely affect the patient’s ability to recover.

References:

• http://www.pallimed.org/2008/07/methadone-methadone-methadone.html • http://www.pallimed.org/2010/01/outpatient-rotations-to-methadone.html • http://www.pallimed.org/2005/07/transdermal-fentanyl-to-methadone.html

Page 11: Pallimed/GeriPal Blogs to Boards - Hospice/Palliative Medicine Board Review 2012 (Q&A + Discussion)

From  Blogs  to  Boards:  AAHPM  2012  Pre-­‐Course  from  the  contributors  of  Pallimed  &  Geripal  

Page 11 of 48  

Notes Question HPM8 Mr. Smith is a 72 year old patient was admitted to hospital from his nursing home for

respiratory distress due to CHF exacerbation. Despite aggressive diuresis attempts, his respiratory distress continued and his urine output remained minimal (~30ml/day). PMH: heart failure, moderate dementia, renal insufficiency Home medications: furosemide 40mg po bid, metoprolol 25mg bid, donepezil 10mg daily, olanzapine 5mg qhs. After a conversation with his son (health care proxy) the patient was "made CMO" (comfort measures only) by the hospitalist service and resident team two days ago. He was then started on a morphine drip “titrate by 1mg as needed for pain or shortness of breath”, his donepezil, olanzapine and diuretics continued, other medications stopped. His intern calls in a panic: “We promised to make him comfortable, that he would die in 2 days, but he is still alive and the family does not know why he is in such pain – even with light touch – crying out & jerking.” What is your recommendation?

a) Stop morphine drip and start fentanyl and lorazepam prn b) Increase morphine and olanzapine c) Increase morphine and add lorazepam prn d) Stop morphine drip and start fentanyl, increase olanzapine

Discussion: Correct answer (a) Key points:

• Opioid neurotoxicity in the setting of renal failure/azotemia is the most likely answer. Morphine metabolites build up disproportionately in the setting of renal failure. Morphine 3-glucoronide is a neurostimulant that can lead to agitated delirium, myoclonus, hyperalgesia, and even seizures. Morphine and hydromorphone are the most common culprits. Morphine 6-glucoronide is a metabolite that is active on the mu-opioid receptor, and thus is not a major player in terms of inducing agitated neurotoxicity.

• Fentanyl does not have the same metabolites and thus has a lower risk of agitated neurotoxicity. Since there are no active metabolites that build up in renal failure, it is the safest of the “pure” opioids for patients on dialysis or who are oliguric. Methadone is another opioid that is nearly ~100% excreted in the stool

• The treatment for this is to rotate off current opioid. Fentanyl is safer option in renal failure.

• Antipsychotics can worsen the symptoms • Benzodiazepines can help treat myoclonus and prevent seizures

References: Robin K Wilson, David E Weissman; Neuroexcitatory effects of opioids: patient assessment, 2nd ed. EPERC# 057 http://www.eperc.mcw.edu/EPERC/FastFactsIndex/ff_057.htm http://www.aahpm.org/apps/blog/?tag=boards Smith, H. S. (2009). Opioid metabolism. Mayo Clinic proceedings. Mayo Clinic, 84(7), 613-24.

Page 12: Pallimed/GeriPal Blogs to Boards - Hospice/Palliative Medicine Board Review 2012 (Q&A + Discussion)

From  Blogs  to  Boards:  AAHPM  2012  Pre-­‐Course  from  the  contributors  of  Pallimed  &  Geripal  

Page 12 of 48  

Notes Question HPM9 BJ, a 65 yo woman with known non-small cell lung cancer, metastatic to her mediastinum,

contralateral lung, and supraclavicular lymph nodes, returns to your clinic for follow-up for her cancer-related pain. She is getting chemotherapy, and has always expressed a desire for ‘the most aggressive’ treatments available for her cancer. She complains of 2 weeks of worsening, midline low back pain. She has noticed difficulty in rising from chairs/toilet, and needed a wheelchair to make it into the clinic area today from the parking garage due to weakness. Examination is notable for an unremarkable back/spine exam, and 4/5 strength bilaterally in her lower extremities both proximally and distally. You obtain a stat MRI which shows a T12 vertebral metastasis and cord compression.

In addition to administering glucocorticoids, then next best step is to:

a) Arrange an urgent radiation oncology consultation for the next day. b) Admit her to the hospital, and arrange a stat radiation oncology consultation. c) Admit her to the hospital, and arrange a stat spine surgery consultation. d) Adjust her pain medications appropriately, and instruct her to contact you immediately

if her pain or disability worsens Discussion: Correct answer is (c) This is a medical emergency.

• Vertebral metastases, putting a patient at risk for cord compression, should be considered in any patient with new back pain and cancer. New or otherwise suspicious back-pain can be evaluated urgently with a non-contrast MRI of the entire spine.

• If patients have neurologic symptoms of LE weakness and/or bladder, bowel dysfunction, it is a medical emergency and patients needs stat imaging, steroids, and intervention. Neurologic deficits, once present, can rapidly progress to permanent paraplegia within 24h.

• The role of steroids + XRT vs steroids + surgery is unclear. A recent trial indicated better outcomes with immediate surgery, especially for patients who came in with severe weakness. 84% of patients vs 54% were ambulatory after treatment course with surgery vs radiation without surgery. Actual practice has not necessarily caught up with this, and will depend on local, institutional resources.

References:

• http://www.pallimed.org/2005/08/surgery-better-than-radiation-steroids.html • http://www.pallimed.org/2008/03/spinal-cord-compression-copd-prognosis.html • http://www.eperc.mcw.edu/EPERC/FastFactsIndex/ff_237.htm • http://www.eperc.mcw.edu/EPERC/FastFactsIndex/ff_238.htm

Page 13: Pallimed/GeriPal Blogs to Boards - Hospice/Palliative Medicine Board Review 2012 (Q&A + Discussion)

From  Blogs  to  Boards:  AAHPM  2012  Pre-­‐Course  from  the  contributors  of  Pallimed  &  Geripal  

Page 13 of 48  

Notes Question HPM10 Mr. G. Da Salva is a 68 year old construction worker who has metastatic non-small cell lung

cancer involving his right femur and pelvis. Medications include: Morphine ER 200mg bid, Morphine IR 30-60mg PO q2 hours prn, and dexamethasone 8mg daily. At rest his pain is well managed, 2/10. However, he fears movement due to severe pain and spends most of the day in his recliner, avoiding showering or changing or helping with the meals. He uses approximately 5 doses daily of 60mg short-acting morphine for this pain but once it starts to work the pain has often spontaneously subsided and he becomes sleepy and confused. Which of the following is LEAST appropriate?

a) Take a short-acting morphine prior to a clustering his activities: showering, changing, fixing a meal.

b) Add sublingual fentanyl 200mcg to take prior to his activities. c) Increase his long-acting morphine to 200mg tid. d) Single-fraction radiation therapy to his pelvis and femur. e) Intrathecal pump with morphine and low-dose bupivacaine.

Discussion: Correct answer is (c) This is an example of incidental pain. It differs from breakthrough pain in that it is associated with movement, and diminishes as soon as the activity ends. The challenge with this form of pain management is that the pharmacology of systemic opioids does not tend to match the timing of this type of pain.

a) Clustering his activities together so that they all take longer may better match the t1/2 of the short-acting morphine, but he will still need to take the medicine approximately 40 minutes prior to starting the activities.

b) Sublingual or buccal fentanyl has a shorter half-life and shorter time to onset than other oral opioids and is a better option.

c) Is the correct answer: Increasing the long-acting morphine is the least appropriate because this will not help the incidental pain and may worsen his confusion when he is not moving around.

d) Single fraction radiation therapy would be very appropriate in this setting and would likely be one of the most preferred interventions as long as he had not previously been irradiated at the site of pain.

e) Intrathecal pain medication delivery is another good option. However, this is an expensive procedure and requires a prognosis of at least 3 months to assure coverage by insurance plans. Because the dose of opioid is a fraction of systemic opioid delivery, its risk of side-effects is lower and is a more effective means of managing incidental pain in the lower back and lower extremities.

References: • http://www.aahpm.org/apps/blog/?p=809 • http://www.pallimed.org/2009/12/poll-results-palliative-care-experience.html • Bruera, E., & Kim, H. N. (2003). Cancer pain. JAMA  : the journal of the American

Medical Association, 290(18), 2476-9. • Chow, E., Harris, K., Fan, G., Tsao, M., & Sze, W. M. (2007). Palliative radiotherapy

trials for bone metastases: a systematic review. Journal of clinical oncology  : official journal of the American Society of Clinical Oncology, 25(11), 1423-36.

• Smith, T. J., Swainey, C., & Coyne, P. J. (2005). Pain management, including intrathecal pumps. Current Pain and Headache Reports, 9(4), 243-248. Current Medicine Group LLC.

Page 14: Pallimed/GeriPal Blogs to Boards - Hospice/Palliative Medicine Board Review 2012 (Q&A + Discussion)

From  Blogs  to  Boards:  AAHPM  2012  Pre-­‐Course  from  the  contributors  of  Pallimed  &  Geripal  

Page 14 of 48  

Notes Question HPM11

Mr. Z is a 87 year old with advanced dementia living in a nursing home. At baseline he cannot recognize family members, is dependent on all ADLs (dressing, toileting, bathing) but does not have urinary or fecal incontinence. He speaks about 1-2 intelligible words per day and he has had progressive loss of ability to ambulate. He is now admitted to the hospital after sustaining a hip fracture from a fall. When discussing treatment options for his hip fracture, his wife asks you how long he likely has to live. Given his current state of health, what would be the most appropriate answer:

a) Given that he does not meet FAST 7C criteria his prognosis is likely greater than 6 months

b) He meets NHPCO Guidelines for hospice eligibility which means he likely has less than a 6 month prognosis

c) Given his advanced dementia and recent hip fracture, his 6 month mortality risk exceeds 50%

d) As with most individuals with advanced dementia, his life expectancy is likely weeks to months

Discussion: Correct answer is (c) References:

a) The FAST scale measures functional status in dementia and consists of 7 major stages split into 16 different sub-stages. Hospice eligibility criteria for dementia are based largely on whether a patient meets or exceeds Stage 7c on the FAST and whether they have at least one complication from their dementia. Unfortunately, these criteria do not accurately predict 6-month survival.

b) The current National Hospice and Palliative Care Organization (NHPCO) guidelines for hospice eligibility are of limited accuracy in predicting death within 6 months. In addition, NHPCO guidelines relies on the FAST staging, which fails to account for the observation that dementia often does not progress in a sequential pattern. The patient is dependent on ADLs (dressing, toileting, bathing) but does not have urinary or fecal incontinence (FAST Stage 6d and 6e). His speech has declined from less than 6 intelligible words per day (7a) to one or less (7b), and he has had progressive loss of ability to ambulate (7C), however since he does not have 6d and 6e, Mr. Z is not considered Fast Stage 7c, rather he is Fast 6C.

c) Is the correct answer: Individuals with advanced dementia that are either hospitalized for either pneumonia or for hip fracture have a very poor prognosis. In one study, six-month mortality for patients with end-stage dementia and hip fracture was 55% compared with 12% for cognitively intact patients.

d) Advanced dementia is a terminal condition; however estimating prognosis is difficult due to the prolonged period of severe functional and cognitive impairment that occurs prior to death. For those with advanced disease who reside in a nursing home, the 6-month mortality rate is 25% with a median survival in one study of only 478 days.

References: § http://www.geripal.org/2009/10/there-is-important-article-in-current.html § www.eprognosis.org § Mitchell SL, Miller SC, Teno JM, Kiely DK, Davis RB, Shaffer ML. Prediction of 6-month

survival of nursing home residents with advanced dementia using ADEPT vs hospice eligibility guidelines. JAMA. Nov 3 2010;304(17):1929-1935.

§ Morrison RS, Siu AL. Survival in end-stage dementia following acute illness. Jama. Jul 5 2000;284(1):47-52.

§ Mitchell SL, Teno JM, Kiely DK, et al. The clinical course of advanced dementia. N Engl J Med. Oct 15 2009;361(16):1529-1538.

Page 15: Pallimed/GeriPal Blogs to Boards - Hospice/Palliative Medicine Board Review 2012 (Q&A + Discussion)

From  Blogs  to  Boards:  AAHPM  2012  Pre-­‐Course  from  the  contributors  of  Pallimed  &  Geripal  

Page 15 of 48  

Notes Question HPM12

Mrs. A is an 88 year old with advanced dementia who lives in a nursing home. She has at baseline some difficulty with eating as she pockets food in her mouth and occasionally coughs after swallowing. She is now hospitalized for an aspiration pneumonia. In addition to the antibiotics she is on in the hospital, her only other medications include HCTZ for hypertension and a baby aspirin. She has never taken a cholinesterase inhibitor . What is the best next step?

a) A trial of both a cholinesterase inhibitors and memantine b) Feeding tube insertion c) Careful hand feeding and good oral care d) Addition of olanzapine to treat her pocketing of food behavior

Discussion: Correct answer is (c)

a) Acetylcholinesterase inhibitors, such as donepezil, galantamine, and rivastigmine, and the N-methyl-D-aspartate (NMDA) antagonist memantine have some evidence for a statistically significant improvement in cognitive, functional, and behavioral outcomes in indivudals with moderate-to-severe dementia. However, these improvements have marginal clinical significance. Adverse events are common with these agents, most commonly nausea, vomiting, and diarrhea. There is no evidence to suggest that they decrease eating problems or risk for aspirations.

b) Placement of PEG tubes often occur after transfer to an acute care facility for eating problems or pneumonia, despite the fact that feeding tubes have not been shown to improve survival for individuals with dementia. There is also no evidence that tube feeding prevents aspiration pneumonia, decreases the risk for pressure ulcers, improves patient comfort, or prolongs life.

c) Oral care has been shown to decrease incidence of pneumonia, number of febrile days, and death from pneumonia in nursing home residents.

d) Antipsychotics have not been shown to improve eating behaviors in dementia. There is a moderate short-term efficacy when treating agitation, serious side effects that include risk of stroke and death limit their clinical use.

References:

• http://www.geripal.org/2011/07/decisions-on-feeding-tubes-in-advanced.html • Finucane TE, Christmas C, Travis K. Tube feeding in patients with advanced dementia:

a review of the evidence. JAMA. Oct 13 1999;282(14):1365-1370. • Yoneyama T, Yoshida M, Ohrui T, et al. Oral care reduces pneumonia in older patients

in nursing homes. J Am Geriatr Soc. Mar 2002;50(3):430-433. • Schneider LS, Tariot PN, Dagerman KS, et al. Effectiveness of atypical antipsychotic

drugs in patients with Alzheimer's disease. N Engl J Med. Oct 12 2006;355(15):1525-1538

Page 16: Pallimed/GeriPal Blogs to Boards - Hospice/Palliative Medicine Board Review 2012 (Q&A + Discussion)

From  Blogs  to  Boards:  AAHPM  2012  Pre-­‐Course  from  the  contributors  of  Pallimed  &  Geripal  

Page 16 of 48  

Notes Question HPM12 – Part 2

The family is concerned that Mrs. A’s aspirations will continue if she continues to be fed by hand in the nursing home. They would like to know about more about the risks of a feeding tube placement.

The most appropriate risk to include in the discussion is:

a) She will have a 1 in 10 chance of a major surgical complication in the perioperative

period. b) She is unlikely to have a tube related complication after the perioperative period c) Once the tube is placed, it would be technically difficult to electively remove the tube d) She will have a 1 in 3 chance of requiring chemical or physical restraints to prevent

tube removal Discussion: Correct answer is (d)

a) Up to one-third experience transient gastrointestinal adverse effects (ie, vomiting,

diarrhea) but major complications like bowel perforations are rare (1%) b) Tube dislodgement, blockage, and leakage are common (4%-11%). One in 5 tube-fed

residents experiencing a tube related complication necessitating a hospital transfer in the year following insertion.

c) Removal of feeding tubes is not technically difficult. An important pearl to know is that if a feeding tube is inadvertently removed, the stoma site will close in a few hours, so put in a Foley catheter to keep it open until a new one can be placed.

d) Over a quarter of tube fed dementia patients are physically restrained after feeding tube placement, and nearly a third are placed on sedating medications to prevent them from pulling out the feeding tube.

References:

• http://www.geripal.org/2011/07/decisions-on-feeding-tubes-in-advanced.html • Teno JM et al. Decision-making and outcomes of feeding tube insertion: a five-state

study. J Am Geriatr Soc. 2011 May;59(5):881-6

Page 17: Pallimed/GeriPal Blogs to Boards - Hospice/Palliative Medicine Board Review 2012 (Q&A + Discussion)

From  Blogs  to  Boards:  AAHPM  2012  Pre-­‐Course  from  the  contributors  of  Pallimed  &  Geripal  

Page 17 of 48  

Notes Question HPM13 A 54 yo man with a 7 month history of metastatic bladder cancer presents to the cancer center’s

palliative care clinic. He complains of low mood, anhedonia, feelings of guilt, shame, and worthlessness most days for the last 2 months. He says, “Of course I’m depressed – who wouldn’t be? I’ve got a cancer that the doctors tell me is terminal. What good am I to my family? They’d be better off without me.” The best next step would be to:

a) Tell the patient that he is depressed and recommend a treatment plan for it. b) Ask your team’s social worker to see the patient for grief counseling. c) Provide emotional support and counseling with the patient that what he is experiencing

is part of the expected adjustment to having a terminal illness. d) Refer the patient to psychiatry for complicated depression.

Discussion: Correct answer is (a)

a) Psychiatric disorders including depression and anxiety disorders occur in only a minority of patients at the end of life. Published numbers have ranged from 10-40%; while higher than the general population, they are not the norm. Persistent low mood, feelings of worthlessness, guilt, and shame are highly suggestive of depression. The patient should be counseled about this and offered treatment

b) While this may end up being appropriate for this patient, his symptomotology is most c/w depression and a) is the better answer.

c) See (a). This is not ‘normal.’ d) Complicated depression is not a diagnosis. He has untreated depression, and HPM

specialists should be able to initiate appropriate therapy! References:

• Rayner L et al. Antidepressants for the treatment of depression in palliative care: systematic review and metaanalysis. Palliat Med. 25(1):36-51. DOI: 10.1177/0269216310380764

• http://www.pallimed.org/2011/02/what-i-learned-at-aahpmhpna-annual.html • Irwin SA. Oral ketamine for the rapid treatment of depression and anxiety in patients

receiving hospice care. JPM. 2010; 13:903-8. http://www.ncbi.nlm.nih.gov/pubmed/20636166

Page 18: Pallimed/GeriPal Blogs to Boards - Hospice/Palliative Medicine Board Review 2012 (Q&A + Discussion)

From  Blogs  to  Boards:  AAHPM  2012  Pre-­‐Course  from  the  contributors  of  Pallimed  &  Geripal  

Page 18 of 48  

Notes Question HPM14

The patient agrees to pharmacologic therapy for his depression, and declines offers of counseling/therapy. Your best estimate is that he has 4-8 weeks to live based on performance status and tempo of decline. Which of the following are appropriate drug approaches for his depression?

a) Methylphenidate b) Ketamine c) Dronabinol d) Sertraline

Discussion: Correct answer are (a) and (b)

a) Is correct: Methylphenidate and other psychostimulants are rapidly-acting, with onset of mood elevation occurring ~immediately (if they are going to be effective at all).

b) Is correct: Ketamine has been described as a depression therapy for decades, albeit one on the margins of accepted medical practice; there is a recent resurgence in interest for its use at life’s end, because its effects are immediate. Patients usually receive a single infusion, which stabilizes mood for weeks at a time. Can be used orally too.

c) Dronabinol has no defined role as an antidepressant. d) Most SSRIs take at least 4 weeks to become effective, which is 50-100% of the

patient’s anticipated survival. While SSRIs are first-line antidepressants in the general population as well as for patients with advanced illness, this is not as true as prognosis shortens.

References:

• Rayner L et al. Antidepressants for the treatment of depression in palliative care: systematic review and metaanalysis. Palliat Med. 25(1):36-51. DOI: 10.1177/0269216310380764

• http://www.pallimed.org/2011/02/what-i-learned-at-aahpmhpna-annual.html • Irwin SA. Oral ketamine for the rapid treatment of depression and anxiety in patients

receiving hospice care. JPM. 2010; 13:903-8. http://www.ncbi.nlm.nih.gov/pubmed/20636166

Page 19: Pallimed/GeriPal Blogs to Boards - Hospice/Palliative Medicine Board Review 2012 (Q&A + Discussion)

From  Blogs  to  Boards:  AAHPM  2012  Pre-­‐Course  from  the  contributors  of  Pallimed  &  Geripal  

Page 19 of 48  

Notes Antidepressant Pop Quiz

I say depression, insomnia, anorexia, nausea – You say: a) Trazodone b) Paroxetine c) Mirtazipine d) Escitalopram I say depression, anxiety, insomnia, neuropathy, You say: a) nortriptyline b) duloxetine c) fluoxetine d) venlafaxine I say activating antidepressants, You say a) fluoxetine b) paroxetine c) buproprion d) citalopram I say depression, anxiety, neuropathic pain, advanced age, You say: a) duloxetine b) nortriptyline c) paroxetine d) mirtazapine Discussion: Correct answers in order are: (c) – (a) – (a, c) – (a)

• Mirtazapine has side effects which include drowsiness and weight gain, plus some antiemetic effects

• TCAs are the only drug class that directly treat all those symptoms (SNRIs – serotonin norepinephrine reuptake inhibitors - like duloxetine and venlafaxine aren’t known to be directly effective for insomnia).

• Paroxetine can be sedating; citalopram more neutral • Duloxetine seems to be better tolerated than TCAs, especially in the elderly. For the

boards, would avoid giving elderly TCAs. Mirtazapine’s role in pain is not well defined, especially compared to SNRIs and TCAs.

Page 20: Pallimed/GeriPal Blogs to Boards - Hospice/Palliative Medicine Board Review 2012 (Q&A + Discussion)

From  Blogs  to  Boards:  AAHPM  2012  Pre-­‐Course  from  the  contributors  of  Pallimed  &  Geripal  

Page 20 of 48  

Notes Question HPM15 Mrs. Phillips is a 91-year-old hospitalized patient who is now actively dying due to end-stage

pulmonary fibrosis and asbestosis. She has been well palliated during the last several months at home where she lived independently, until she developed a pneumonia and was hospitalized. Her home medications had not been adjusted in over six weeks. This included: albuterol and atropine nebulizers, dexamethasone 2mg every morning, 25mcg/hour fentanyl patch for dyspnea, oxycodone concentrate (20mg/ml) 10mg q2 hours prn dyspnea or pain, senna and Colace. She is on day 7 of oral antibiotics for presumed pneumonia. She is on oxygen 6 liters via nasal cannula. Her last bowel movement was yesterday, and her urine output has been good (250ml or more daily.) Yesterday she was still oriented, between periods of increasing fatigue and sleep. She showed signs of mottling and new secretions causing respiratory rattle. A scopolamine patch 1.5mg was started for her increased secretions. You are called by the resident who explains to you that this morning Mrs. Phillips is now agitated, moaning, and even thrashing at times. This is causing family and floor nurses distress. He asks you for advice. Which of the following is appropriate?

a) Stop scopolamine b) Start lorazepam c) Increase the fentanyl d) Stop the fentanyl e) Counsel family about the inevitability terminal delirium f) Order soft restraints

Discussion: (a) is correct Delirium is a common condition at the end-of-life. It often is considered “terminal” even if reversible, however. Terminal delirium should be considered a diagnosis of exclusion or even one made in hindsight. While conducting a battery of exhaustive tests to evaluate the cause is not usually appropriate or necessary, causes of delirium should be addressed if possible. The most common causes of delirium in this setting remain constipation, urinary retention, medications, infection, electrolyte abnormalities. Constipation, urinary retention can be ruled out in this patient.

a) With this patient, the addition of scopolamine is the most likely cause. This is a tertiary amine anticholinergic agent, and commonly causes confusion in the elderly.

b) Lorazepam is not the best option for delirium; neuroleptics, in addition to treating the underlying cause (if feasible) are appropriate.

c) Fentanyl, on the other hand, is less likely the cause. She has been on a stable dose since home and was previously tolerating it well. Stopping the fentanyl will likely increase delirium, dyspnea and withdrawal symptoms. Increasing the fentanyl, similarly, is unlikely to address the agitation – unless the patient has been responding to breakthrough oxycodone.

d) As above e) Family members should be comforted, but not that it is an inevitable part of dying. f) Restraints should be avoided.

References: • http://www.geripal.org/2010/06/ny-times-article-on-delirium.html • http://www.eperc.mcw.edu/EPERC/FastFactsIndex/ff_001.htm

Page 21: Pallimed/GeriPal Blogs to Boards - Hospice/Palliative Medicine Board Review 2012 (Q&A + Discussion)

From  Blogs  to  Boards:  AAHPM  2012  Pre-­‐Course  from  the  contributors  of  Pallimed  &  Geripal  

Page 21 of 48  

Notes Question HPM16

Mr. J is 58 year old diagnosed with ALS 6 months ago. He is referred to your clinic by his primary care doctor to help discuss options to treat a progressive weight loss. He currently lives alone in an apartment, is independent of ADLs although he has been having difficulty feeding himself due to proximal arm weakness. He complains that he occasionally bursts out crying or laughing, but denies feeling depressed. His forced vital capacity (FVC) has remained at 70% for the last 3 months. The best next step to help treat his progressive weight loss?

a) Riluzole b) PEG Placement c) Mobile arm supports and modified cutlery d) Non Invasive Positive Pressure Ventilation (NIPPV)

Discussion: (c) is the correct answer

a) Riluzole is the only available disease-modifying therapy for ALS. Based on clinical trials, riluzole likely prolongs median survival in patients with ALS by 2-3 months compared to patients taking placebo.(1) It does little to improve functional outcomes or bulbar symptoms. There is no evidence to suggest that it is beneficial for weight loss.

b) PEG placement should be discussed with any individual diagnosed with ALS, although attempt to reverse other common reasons for weight loss is warranted before PEG placement. Ideally, PEG tubes should be placed before FVC falls below 50%. While PEG may be indicated for this patient in the future, currently C is the best option.

c) Individuals living with ALS may have difficulty with the mechanics of both cooking and putting food from the plate to the mouth. This may often contribute to weight loss, especially for those individuals living alone. Occupational therapy may help maintain adequate nutrition by supplying devices such as mobile arm supports and modified cutlery.

d) NIPPV confers a survival benefit and improves quality of life in patients with normal or moderately impaired bulbar function, although it does not improve weight

References:

• Miller RG, Mitchell JD, Lyon M, Moore DH. Riluzole for amyotrophic lateral sclerosis (ALS)/motor neuron disease (MND). Cochrane Database Syst Rev. 2007 Jan 24;(1):CD001447.

• Mitsumoto H, Rabkin, JG. Palliative Care for Patients With Amyotrophic Lateral Sclerosis “Prepare for the Worst and Hope for the Best”. JAMA. 2007;298(2):207-216

Page 22: Pallimed/GeriPal Blogs to Boards - Hospice/Palliative Medicine Board Review 2012 (Q&A + Discussion)

From  Blogs  to  Boards:  AAHPM  2012  Pre-­‐Course  from  the  contributors  of  Pallimed  &  Geripal  

Page 22 of 48  

Notes Question HPM17

Mr G is a 74-year-old nursing home resident with coronary artery disease and end-stage renal failure (eGFR of 12). He is considering starting treatment with dialysis but would like to know more about what life will be like after starting dialysis. What would be the most accurate statement in regards to his prognosis

a) His functional status is likely to improve with renal replacement therapy b) His functional status is likely to be maintained at his pre-dialysis level c) He is unlikely to have significant symptom burden if he elects not to initiate dialysis d) The majority of nursing home residents die within one year of starting dialysis

Discussion: (d) is the correct answer

a) Based on a NEJM paper (1) that linked dialysis registry data to activities of daily living measures reported by nursing homes in 3,702 patients, patients similar to Mr. G did poorly. Within 3 months after the start of dialysis, 61% of the nursing home residents had died or had a decrease in functional status as compared with their functional status before dialysis. Only 39% had the same functional status that they had before dialysis. By 12 months, almost all (87%) nursing home residents had died or had a decrease in functional status after starting dialysis.

b) See a above c) The last month of life for individuals who elect not to undergo renal replacement

therapy is associated with relatively high symptom burden, similar to that of advanced cancer.(2) Common symptoms include lack of energy, itching, feeling drowsy, shortness of breath, difficulty concentrating, pain, lack of appetite, and swelling of arms/legs. Therefore, clinicians should attend to these symptoms as aggressively as they do for patients with advanced cancer. We don’t know though whether symptom burden is improve or worsened for elderly patients with multiple chronic conditions who do elect for renal replacement therapy.

d) In the NEJM cited above, 58% of these nursing home residents had died 1 year after initiating dialysis

References:

• http://www.geripal.org/2009/10/how-should-we-counsel-frail-nursing.html • http://www.geripal.org/2010/09/dying-without-dialysis.html • Kurella Tamura M, Covinsky KE, Chertow GM, Yaffe K, Landefeld CS, McCulloch

CE. Functional status of elderly adults before and after initiation of dialysis. N Engl J Med 2009;361:1539-1547

• Murtagh FE, et al. Symptoms in the Month Before Death for Stage 5 Chronic Kidney Disease Patients Managed Without Dialysis. J Pain Symptom Manage. 2010 Sep;40(3):342-52.

Page 23: Pallimed/GeriPal Blogs to Boards - Hospice/Palliative Medicine Board Review 2012 (Q&A + Discussion)

From  Blogs  to  Boards:  AAHPM  2012  Pre-­‐Course  from  the  contributors  of  Pallimed  &  Geripal  

Page 23 of 48  

Notes Question HPM18

George Condi is a 68 y/o male is admitted to ICU for respiratory crisis and found to have renal cell carcinoma with a 13 cm mass in the R upper abdomen. He has severe pain, and dyspnea with large R sided pleural effusion. With drainage of effusion his dyspnea is improved; a tunneled pleural catheter is placed, and he is discharged to home hospice with a PPS of 50. The next day his wife calls saying she can’t manage the catheter and she is in tears because his pain is 6/10 and he is more short of breath. “You promised me it wouldn’t be like this!” She wants to take him to the emergency room for IV furosemide and a pulmonologist visit. The best approach is to:

a) Arrange for a hospice nurse to meet the patient in the emergency room to disenroll him from hospice

b) Set up in home continuous care to manage his catheter c) Immediately prepare a respite stay d) Admit the patient to a qualified skilled nursing facility for General Inpatient stay for

pain control Discussion: (d) is best

a) The family needs are symptom management and training about catheter care. Both are best provided in a controlled environment, not in the emergency room. This case represents a failure in transitions of care, and disenrollment from hospice will only lead to one more transition

b) Continuous care could be a reasonable choice if the issue were only the catheter. The requirement for skilled care is the same as for gip and it is a good option for a patient who really does not want to be in a facility but in this case the patient’s intensive symptom need may need 24 hour nursing care and continuous care is not entirely provided by nursing level care

c) Respite is for the benefit of the family and is generally a planned event, it is also appropriate to use in cases of caregiver breakdown when there is not a skilled care need requirement for the patient

d) In this case, the patient has severe dyspnea and pain and is requiring both catheter drainage of an effusion and rapid titration of opioids to control his symptoms in a manner that his care givers are not capable of providing currently. GIP is to provide skilled care for the patient that cannot be provided in the home. Documentation for GIP based on pain must include:

• Frequent evaluation • Frequent medication adjustment • Aggressive interventions to control the pain

References:

• http://www.aahpm.org/apps/blog/?p=1133

Page 24: Pallimed/GeriPal Blogs to Boards - Hospice/Palliative Medicine Board Review 2012 (Q&A + Discussion)

From  Blogs  to  Boards:  AAHPM  2012  Pre-­‐Course  from  the  contributors  of  Pallimed  &  Geripal  

Page 24 of 48  

Notes Question HPM19

George is admitted to GIP status in a skilled nursing facility with 24 hour RN availability. He has had a marked decline since he was seen 2 days ago. The hospice nurse is asking whether the plan should be to send him back home after the symptoms are controlled. The social worker doesn’t want to bring that up because it might upset the wife and because it might give George false hope. The entire Interprofessional group thinks he might die in the next week or two You reply that:

a) Since he’ll likely die in 7-10 days, it will be fine to continue on General Inpatient Status for imminently dying criteria so discharge discussions don’t need to be raised

b) Due to the wife’s burden of caregiver distress, the patient will be maintained on General Inpatient Status for caregiver breakdown so discharge discussions don’t need to be raised

c) Once admitted to General Inpatient Status, one of the goals must be transition to a lower level of care

d) Since General Inpatient status should only last 7 days, discharge discussions will start after the first 3 days to let the family have some relief.

Discussion: Correct answer is (c)

a) There is no GIP status for ‘imminently dying’. There must be some symptom that requires management

b) CMS has clarified that GIP should only be used based on the patient condition and should not be used due to caregiver “breakdown”. (CMS Quarterly Provider Update April 2007, http://www.cms.hhs.gov/quarterlyproviderupdates/downloads/cms1539p.pdf)

c) Direct wording from quarterly. The goal may not be achievable, but needs to be a part of planning and discussion.

d) There is no specified time limit to GIP status, although some fiscal intermediaries do appear to increase audits after the first 7 days

References:

• http://digital.ipcprintservices.com/publication/?i=93241 (login required using aahpm membership id)

• Medicare Benefit Policy Manual Chapter 9 - Coverage of Hospice Services Under Hospital Insurance 40.1.5 - Short-Term Inpatient Care pp15 and 16/32 http://www.cms.gov/manuals/downloads/bp102c09.pdf

Page 25: Pallimed/GeriPal Blogs to Boards - Hospice/Palliative Medicine Board Review 2012 (Q&A + Discussion)

From  Blogs  to  Boards:  AAHPM  2012  Pre-­‐Course  from  the  contributors  of  Pallimed  &  Geripal  

Page 25 of 48  

Notes Question HPM20 Mrs. Tagliatelli is a 76 year old Italian immigrant and widow who has not missed a day of mass

in her adult life until this past month. She comes to see her primary care physician in clinic because she missed mass, asking whether she should get hospice. She has heart failure, mild hypertension, and sleep apnea. She has noted that over the last month, her legs are more swollen and she is having increased difficulty walking to church and the grocery store. She still keeps an impeccable home, managing her housecleaning herself, but now is sitting down for a longer period of time after carrying the vacuum up and downstairs. She is also able to maintain her daily rituals of reading the NYTimes Health and Travel sections, cooking three small meals each day. She no longer wishes to return to hospital, and has not been admitted since her myocardial infarction 5 years ago, which preceded her diagnosis of heart failure. At that time, she had a successful resuscitation and wishes to remain full code. She uses CPAP at night for her sleep apnea, but otherwise does not require oxygen. She also tells you that because she lives alone, she keeps a gun in her home for self-protection. Her home medications include: Furosemide 10mg BID, Atenolol 50mg daily, lisinopril 10mg daily, simvastatin 5mg daily, aspirin 81mg daily. She also has nitroglycerine 0.4mg sl prn (which she has not used since her MI), and acetaminophen 325mg which she takes “once in a while for an ache.” Why would this patient not be admitted to hospice?

a) She is full code. b) She lives alone. c) She has greater than a six-month prognosis. d) She is not homebound. e) She has firearms in the home.

Discussion: answer is (c) Take home points:

• Prognostication for heart failure is one of the more scrutinized and difficult under the hospice guidelines. Her current NYHA class is 2, due to increased symptoms with activity. In addition, her medical management can still be adjusted, likely with good response. General guidelines for heart failure include hospitalization within the last 6-months to a year, dyspnea with minimal exertion or at rest. Since she still is independent in all ADLs including thorough house-cleaning, without becoming dyspneic, it is not reasonable to say she has a <6mo prognosis.

• While to qualify for visiting nursing services, a patient must be homebound, no such requirement exists for hospice.

• Additionally, patients may choose to be full code on hospice. Medicare does not require a 24 hour caregiver to be present in order for the patient to receive hospice services. While firearms are concerning for patient and staff safety, they do not prevent hospice admission. Some hospices have adopted a stance to request patients to have their firearms in locked safe or gun closet.

References: See Q21

Page 26: Pallimed/GeriPal Blogs to Boards - Hospice/Palliative Medicine Board Review 2012 (Q&A + Discussion)

From  Blogs  to  Boards:  AAHPM  2012  Pre-­‐Course  from  the  contributors  of  Pallimed  &  Geripal  

Page 26 of 48  

Notes Question HPM21

A couple of years and hospitalizations later, Mrs. Tagliatelli was admitted to hospice. At the time of admission to hospice, she was breathless with minimal exertion. Neighbors and members of her church visited her often offering her food, company, and rides to church. She required oxygen all the time. Even with this, at the time of admission to hospice, she experienced constant dyspnea. Her cardiac medications were continued, morphine ER and IR were added for her dyspnea. After six months on hospice, she is now well palliated, especially since she has been able to have her medications as prescribed and no longer spaces out her medications in order to make them last. However, she continues to require help from her friends and neighbors, and oxygen with minimal activity. She fell once and required a trip to the emergency department. You go to see her for recertification visit. What do you write in your recertification note?

a) She meets criteria for recertification because her prognosis remains 6-months or less. b) She does not meet criteria for recertification because she has not shown decline in her

condition. c) She does not meet criteria for recertification because her last hospitalization was

unrelated to her hospice diagnosis Discussion (a) is the correct answer Take home points:

• Some intermediaries recommend the demonstration of decline in clinical condition for a patient to be recertified, and while this is helpful is the recertification process, it is not a CMS requirement. According to Medicare guidelines, the only requirement for hospice is that a patient’s prognosis is 6 months or less.

• Documentation of a hospitalization can also help qualify a patient for hospice. The cause of hospitalization does not need to be related to the hospice diagnosis.

References:

• https://www.cms.gov/Hospice/Downloads/HospiceFace-to-FaceGuidance.pdf • http://www.geripal.org/2011/02/hospice-face-to-face-ftf-encounters-for.html

Page 27: Pallimed/GeriPal Blogs to Boards - Hospice/Palliative Medicine Board Review 2012 (Q&A + Discussion)

From  Blogs  to  Boards:  AAHPM  2012  Pre-­‐Course  from  the  contributors  of  Pallimed  &  Geripal  

Page 27 of 48  

Notes Question HPM22a, 22b

A young man was recently in a motor vehicle collision where he suffered a massive head injury and multi-trauma. He was resuscitated and survived in the ICU with a ventilator, continuous hemodialysis, and multiple pressors for the past 2 days, but now is declining, and he is not expected to survive this hospitalization. You receive a palliative care consult to help with the ventilator withdrawal. You head down to the unit and the nurse comes to you and says “I am not sure you should talk with the family – the organ procurement agency has just visited to discuss organ donation after cardiac death, and the family want to donate his organs – his liver and lungs may be transplantable.” What is the best next step?

a) Thank the nurse, and back out of the consult b) Talk with the family about the patient, their grief, and counsel them about comfort care

after cessation of life-support. c) Ask the attending physician of record who is going to manage the patient’s comfort

care after cessation of life-support. d) Work with the family to help them realize this will only prolong the patient’s

suffering. Part B A day later, the patient’s HCV test comes back positive and he is no longer a viable DCD candidate. The ICU attending asks you to ‘take care of the treatment withdrawal’. The family is very disappointed, and indicates their only goal at this point is for a comfortable death, without ‘prolonging this any longer.’ His only symptom-directed med is intermittent fentanyl bolus (700mcg the last 24h). He is unresponsive on the vent, without any spontaneous movement. The best next step is to:

a) Recommend rapidly stopping all life support including CRRT, ventilator, and pressors over the next hour or so, and starting a fentanyl and lorazepam infusion to keep the patient sedated.

b) Recommend staggering withdrawal of life support over a couple days including stopping CRRT and pressors now in the hopes that the patient dies on the ventilator.

c) Discuss with the family different approaches to life-support withdrawal. d) Switch the patient from fentanyl to morphine boluses as you extubate him, as

morphine is more effective for air-hunger. Discussion: Correct answers are Part A(b) Part B (c) Part A Take Home Points

• Donation after cardiac death is an important, and growing, public good, as it expands the pool of potential organ donors: it saves lives. Palliative consultants should support DCD programs, and palliative consultation alongside DCD is possible and in some institutions the standard of care. For most families, the real concern is not in prolonging suffering but the disappointment which can occur if the dying patient becomes ineligible to donate organs. DCD practices and the role palliative care consultants can play in them are spelled out nicely in this Fast Fact: http://www.eperc.mcw.edu/EPERC/FastFactsIndex/ff_242/htm.

• While C is an important question as the palliative consultant may be asked to manage

the patient’s comfort care after extubation in the window during which if the patient dies he will be able to donate his organs, that is a secondary concern right now to doing the good work of palliative care – meeting a patient and family and helping to meet their emotional and informational needs.

Part B Take Home Points

• A study in 2007 showed that family satisfaction was greater with a stuttered/prolonged

Page 28: Pallimed/GeriPal Blogs to Boards - Hospice/Palliative Medicine Board Review 2012 (Q&A + Discussion)

From  Blogs  to  Boards:  AAHPM  2012  Pre-­‐Course  from  the  contributors  of  Pallimed  &  Geripal  

Page 28 of 48  

approach to discontinuing therapies if the patient had been in the ICU for more than 4 days, but when the patient was in for 3days or less, a stuttered approach lowered satisfaction. Even with those clearly stated goals, careful discussions with the family are best about the options for cessation of life-prolonging treatments as families may have anticipated concerns about stopping certain treatments (fluids, tube feeds in particular). Given the modest evidence of improved satisfaction with staggered withdrawal, dicussing with family is the best policy.

• Answer a) may end up being the best approach; however simply starting a fentanyl infusion is secondary to providing/ensuring adequate rapid, boluses of symptom medications are available around the time of extubation.

• No opioid has been established as being superior over another for air hunger. In addition, the patient is in renal failure and morphine may accumulate whereas fentanyl will not; on the other hand, he is likely to live just a short amount of time making that a moot point generally. Morphine is more economical and familiar; on the other hand one knows how much fentanyl this patient has needed as a starting point. On top of this, this comatose patient probably cannot experience air hunger; the indication for opioids really is to palliate labored respirations. All this is to say d is both unnecessary but not contraindicated, but not for the reason stated.

References:

• http://www.pallimed.org/2008/10/stuttered-treatment-withdrawal-in-icu.html • http://cases.pallimed.org/2009/01/coordination-of-care-for-people-at-end.html • http://www.pallimed.org/2006/03/terminal-patients-in-icu-and-organ.html • Gerstel E, Engelberg RA, Koepsell T, & Curtis JR. (2008) Duration of withdrawal of

life support in the intensive care unit and association with family satisfaction. American journal of respiratory and critical care medicine, 178(8), 798-804. PMID: 18703787

• Revelly JP, Imperatori L, Maravic P, Schaller MD, & Chioléro R. (2006) Are terminally ill patients dying in the ICU suitable for non-heart beating organ donation?. Intensive care medicine, 32(5), 708-12. PMID: 16534569

• Lynn, J. (2001-2-21) Serving Patients Who May Die Soon and Their Families: The Role of Hospice and Other Services. JAMA: The Journal of the American Medical Association, 285(7), 925-932. DOI: 10.1001/jama.285.7.925

• Vent withdrawal FF: http://www.eperc.mcw.edu/EPERC/FastFactsIndex/ff_033.htm • http://www.eperc.mcw.edu/EPERC/FastFactsIndex/ff_034.htm • DCD FF: http://www.eperc.mcw.edu/EPERC/FastFactsIndex/ff_242/htm • Anoxic Brain Injury FF:

http://www.eperc.mcw.edu/EPERC/FastFactsIndex/ff_234.htm

Page 29: Pallimed/GeriPal Blogs to Boards - Hospice/Palliative Medicine Board Review 2012 (Q&A + Discussion)

From  Blogs  to  Boards:  AAHPM  2012  Pre-­‐Course  from  the  contributors  of  Pallimed  &  Geripal  

Page 29 of 48  

Notes Question HPM23

A 47 year old woman with a severe, idiopathic, dilated cardiomyopathy is receiving hospice care at home. She is ineligible for cardiac transplantation or a ventricular assist device. She has mild resting dyspnea but becomes severely dyspneic after just a few steps of ambulation. Her nurse measures her resting and ambulatory oxygen saturation while breathing ambient air: it is 96 and 92%, respectively. The patient is taking digoxin, bumetamide, hydralazine, isosorbide dinitrate, albuterol MDI, warfarin, senna, and clonzepam. The patient requests home oxygen therapy to help alleviate her breathlessness. The best response is:

a) Order home oxygen therapy for the patient b) Initiate lorazepam prn for dyspnea c) Recommend use of a hand-held fan and prn morphine for her dyspnea d) Request that the patient see her cardiologist for further optimization of her heart failure

meds Discussion: Correct answer is (c)

a) Home oxygen therapy is not recommended as first-line treatment for dyspnea in non-hypoxic patients. It has been shown to be equivalent to ‘sham’ delivery of ambient air via nasal cannula. While there is a role for it even in normoxic patients (it ‘works’, just no better than ambient air), it is not first-line.

b) She is already on a benzodiazepine, and benzodiazepines are generally considered 2nd line agents to opioids

c) Hand-held fans have been shown to improve dyspnea, and there is professional consensus that opioids are first-line agents for the symptomatic relief of refractory dyspnea that is not responding to treatment of the underlying cause.

d) While there is a role for this strategy, the patient is already on multiple heart failure medications which clearly are not sufficient to palliative her dyspnea, and so c is the best answer.

References:

• http://www.pallimed.org/2010/09/rct-of-oxygen-vs-room-air-delivered-by.html • Viola R et al. The management of dyspnea in cancer patients: a systematic review.

Supp Care Cancer. 2008; 16:329-337. • Galbraith S. Does the use of a handheld fan improve chronic dyspnea? A randomized,

controlled, crossover trial. J Pain Symptom Manage. 2010 May;39(5):831-8.

Page 30: Pallimed/GeriPal Blogs to Boards - Hospice/Palliative Medicine Board Review 2012 (Q&A + Discussion)

From  Blogs  to  Boards:  AAHPM  2012  Pre-­‐Course  from  the  contributors  of  Pallimed  &  Geripal  

Page 30 of 48  

Question HPM24 Mr. L is a 52-year-old homeless man. One week ago, he was admitted to the ICU with respiratory distress

and was intubated. A chest CT scan revealed a large necrotic mass filling the right hemithorax, obliterating the right and narrowing the left mainstem bronchi. Sputum cytology confirmed a diagnosis of non-small cell lung cancer. Oncology states that there is no role for chemotherapy or radiation unless he could be weaned off the ventilator, which was considered doubtful in the setting of his airway obstruction. Mr. L is unable to participate in medical decision-making. The patient’s mother, who is the authorized decision maker, meets with the palliative care team to discuss prognosis and treatment options, including withdrawal of life-sustaining treatments. The mother is adamant that all life-sustaining measures be continued despite a previous discussion that Mr. L’s disease severity will prevent him from ever leaving the ICU, let alone the hospital. Mr. L’s mother expresses hope that, despite the physician’s prediction, a miracle will occur that will allow her son to leave the hospital. The next best step is to:

a) Schedule another family meeting to reiterate the prognosis of his current condition and the likelihood of recovery

b) Involve an ethics committee as the mother’s belief in a miracle is far from a societal norm c) Tell the mother that hope for a miracle is unreasonable, but that she could still hope that her son is

comfortable d) Ask the mother about her spiritual beliefs and how it influences her decision

Discussion: Correct answer is (d) a) Although it would be reasonable to again discuss prognosis, the mother’s hope in a miracle can be

considered at least an acknowledgement that she heard the original prognostic information. Furthermore, spiritual beliefs, including that of a belief in miracles, may trump a physician’s opinion of prognosis despite adequate communication of prognosis.(1) One study of surrogates of incapacitated critically ill patients at high risk for death found that only 2% based their views of prognosis solely on the physician’s prognostic estimate.(2) Rather, these surrogates used a combination of sources including knowledge of the patient’s intrinsic qualities and will to live; their observations of the patient; their own observations and beliefs in the power of their support and presence, and optimism, intuition, and faith (For 20% of surrogates, a faith in God overrode any other source of prognostic information).(1) While scheduling another meeting may be appropriate, the ‘problem’ in this scenario is not lack of DATA, and D is the better answer.

b) According to a 2007 survey performed by the Pew Forum on Religion and Public Life, the majority of Americans believe in miracles with little difference based on the respondent’s age. In another survey, most public respondents (57.4%) believed that divine intervention from God could save a person even if the physician told them ‘‘futility had been reached.’’(3) Significant differences in belief in miracles were noted in this study between health care professionals and the general public. The majority of public respondents (61.3%) believed that a person in a persistent vegetative state could be saved by a miracle, although only a minority of trauma professionals had the same belief.

c) Without knowing more about the mothers spiritual convictions and having a good understanding of what a “miracles means for her”, and reframing of her hope would be premature and could be perceived as condescending.(1)

d) Most individuals would like physicians to ask about their spiritual/religious beliefs.(1) In addition, patients who report that their spiritual needs are supported by the medical team are more likely to receive hospice care than those who report their spiritual needs were unsupported.(2)

References: • http://www.geripal.org/2011/06/lessons-i-learned-by-examining-miracles.html • Widera EW, Rosenfeld KE, Fromme EK, Sulmasy DP, Arnold RM. Approaching patients and

family members who hope for a miracle. J Pain Symptom Manage. 2011 Jul; 42(1):119-25. • Boyd EA, Lo B, Evans LR, et al. ‘‘It’s not just what the doctor tells me:’’ factors that influence

surrogate decision-makers’ perceptions of prognosis. Crit Care Med 2010;38:1270e1275. • Jacobs LM, Burns K, Bennett Jacobs B. Trauma death: views of the public and trauma

professionals on death and dying from injuries. Arch Surg 2008; 143:730e735. • Balboni TA, Paulk ME, Balboni MJ, et al. Provision of spiritual care to patients with advanced

cancer: associations with medical care and quality of life near death. J Clin Oncol 2010;28:445e452

Page 31: Pallimed/GeriPal Blogs to Boards - Hospice/Palliative Medicine Board Review 2012 (Q&A + Discussion)

From  Blogs  to  Boards:  AAHPM  2012  Pre-­‐Course  from  the  contributors  of  Pallimed  &  Geripal  

Page 31 of 48  

Notes Question HPM25 Omar Johnson is a 64 year old man with cryptogenic cirrhosis in multiorgan system failure in

your hospital’s ICU. He is ventilated, unresponsive, and on vasopressors. You and the ICU team agree his chances for surviving this hospitalization are minimal. He has no advance directive. You participate in an ICU family care conference with his wife (his legal decision maker based on state law), 2 sisters, and 3 adult sons. They are told he is dying with minimal chance of survival. His sons say they do not think the patient would want to die ‘like this – on machines,’ and describe several conversations with the patient to support that preference. His wife seems to agree with that, but also says, “I can’t give up on him. I can’t have that on my shoulders – I’ll always wonder if I did the right thing.” The best, next response would be:

a) Request ethics consultation b) Along with the ICU physician, suggest to the family that you make the decision on

behalf of the patient yourselves, to transition the patient to comfort-care. c) Ask the family to focus on what the patient himself would prefer in these

circumstances. d) Express to the family acknowledgment of the emotional difficulty of this, and

recommend another meeting the next day. Discussion: Correct answer is (b)

a) Be wary of it as a board question answer. Ethics consultations have been shown to help with conflict although it wouldn’t be the “best, next response” in this situation.

b) Making recommendations for what you believe to be the best plan of care is critically important in these situations. Surrogate decision making is uniquely traumatizing to family members (beyond routine bereavement); when a patient’s preferences are clear, physicians should clearly articulate an appropriate plan of care and not force family members into feeling they are responsible for a patient’s death.

c) Generally a good idea – but they already have acknowledged those ‘facts’; the issue here is more comfort with decision-making roles and guilt.

d) A 2nd best option to 2 References:

• http://www.pallimed.org/2011/03/trauma-of-surrogacy.html

Page 32: Pallimed/GeriPal Blogs to Boards - Hospice/Palliative Medicine Board Review 2012 (Q&A + Discussion)

From  Blogs  to  Boards:  AAHPM  2012  Pre-­‐Course  from  the  contributors  of  Pallimed  &  Geripal  

Page 32 of 48  

Notes Question HPM26 Mrs. Hassad is a 83 year old retired professor from who is being evaluated for a hospice admission. Her 4 sons live in

adjacent homes with their families. She has metastatic breast cancer with bone, liver, and brain. Because of her underlying renal failure and moderate heart failure, she will not be receiving chemotherapy and her physician had arranged home hospice services now that she has completed palliative radiation. She is alert, oriented. The hospice intake nurse calls you because the family and patient state that she does not want to know anything about his diagnosis or severity of illness. Mrs. Hassad’s son tells the nurse not to speak with the patient about her prognosis, her illness, or about code status. Instead, she asks that you speak with her son about these matters. You are at the home with the nurse because she does not know how to get her to sign the paperwork to enroll in hospice. What do you do after confirming with Mrs. Hassad that she does not want to be involved in signing papers or knowing details of his medical condition, and would rather that you speak with her son?

a) Explain to the son that you must gain consent from Mrs. Hassad in order to enroll her in hospice in respect of the principle of autonomy.

b) Invoke the health care proxy and have Mrs. Hassad’s son sign the paperwork to enroll in hospice. c) Have the son sign the paperwork for hospice since Mrs. Hassad made the autonomous decision to defer

decisions to her son. d) Refuse hospice enrollment for the patient since she is unwilling to accept to address her diagnosis and

prognosis. e) Clarify to the patient that it is her responsibility to make the decision, based on autonomy, and to avoid trauma

of surrogacy in her son. f) Teach the nurse that she should not have questioned the son’s request because that was disrespectful to their

culture. Discussion: (c) is the correct answer

• Patients can make the autonomous decision to know or not to know information. As outlined in the SPIKES protocol for giving bad news, the Invitation is to ask how much a patient wishes to know. Should patients choose not to be informed they should know that then consents to procedures and medical care must also be deferred to the person they request disclosure to.

• Healthcare proxy is invoked only when patients lack the capacity to make medical decisions. Here Mrs. Hassad has capacity but chooses to have her son make decisions on her behalf.

• Hospice enrollment does not require a patient’s acceptance of his or her disease and prognosis. • Research shows that most people, regardless of culture or country of origin, wish to have medical information

disclosed to them. However, as age and illness advance, patients are more inclined to request less disclosure. One should not assume desire or lack of desire to be involved in medical decision-making and disclosure of information based on culture, religion or country of origin.

References:

§ http://www.pallimed.org/2011/03/trauma-of-surrogacy.html § Oncotalk. (n.d.). Giving Bad News., http://depts.washington.edu/oncotalk/learn/modules/Modules_02.pdf § Wendler, D., & Rid, A. (2011). Systematic review: the effect on surrogates of making treatment decisions for others.

Annals of internal medicine, 154(5), 336-46. Retrieved from http://www.annals.org/cgi/content/abstract/154/5/336 § Elkin, E. B., Kim, S. H. M., Casper, E. S., Kissane, D. W., & Schrag, D. (2007). Desire for information and

involvement in treatment decisions: elderly cancer patients’ preferences and their physicians' perceptions. Journal of clinical oncology  : official journal of the American Society of Clinical Oncology, 25(33), 5275-80.

§ Baile, W. F. (2000). SPIKES--A Six-Step Protocol for Delivering Bad News: Application to the Patient with Cancer. The Oncologist, 5(4), 302-311.

§ Elkin, E. B., Kim, S. H. M., Casper, E. S., Kissane, D. W., & Schrag, D. (2007). Desire for information and involvement in treatment decisions: elderly cancer patients’ preferences and their physicians' perceptions. Journal of clinical oncology  : official journal of the American Society of Clinical Oncology, 25(33), 5275-80.

§ Asghari, F., Mirzazadeh, A., & Fotouhi, A. (2008). Patients’ preferences for receiving clinical information and participating in decision-making in Iran. Journal of medical ethics, 34(5), 348-52.

§ Bushnaq, M. (2008). Palliative care in Jordan: culturally sensitive practice. Journal of palliative medicine, 11(10), 1292-3. Mary Ann Liebert, Inc. 140 Huguenot Street, 3rd Floor New Rochelle, NY 10801-5215 USA.

Page 33: Pallimed/GeriPal Blogs to Boards - Hospice/Palliative Medicine Board Review 2012 (Q&A + Discussion)

From  Blogs  to  Boards:  AAHPM  2012  Pre-­‐Course  from  the  contributors  of  Pallimed  &  Geripal  

Page 33 of 48  

Notes Question HPM27 Dr. L is a 44 year old palliative care fellow about to complete two months of a busy inpatient

consult rotation. You notice that over the last week she has become detached and disengaged when talking with patients and their family members. The fellow acknowledges feeling tired and drained most of the time, as well as having difficulty falling asleep. She also confides in you a personal sense of failure and self-doubt. The most appropriate interventions at this time is

a) Recommend she see her primary doctor to discuss SSRI therapy b) Recommend she try bright light therapy c) Refer for a transient mirrectomy d) Recommend an educational program in mindful communication

Discussion: Correct answer is (d)

a) The fellow does not meet DSM-IV criteria for depression, although she does have some suggestive symptoms. Further exploration would be a correct answer, but starting an SSRI would not.

b) There is no evidence that bright light therapy is helpful for symptoms of burnout. There is some evidence of a small benefit for depressive symptoms though.

c) Transient mirrectomy is a fictional treatment described by Brad Stuart in an April fools day GeriPal post. It reportedly is a non-invasive method of numbing brain centers that may induce clinicians to identify with pain and suffering to a disabling degree. Sure sounds nice.

d) The fellow has symptoms suggestive of burnout. Burnout encompasses 3 domains: feelings of emotional exhaustion, cynicism or depersonalization, and a low sense of personal accomplishment. The criterion standard for measuring burnout is the Maslach Burnout Inventory (MBI). There is scant high-quality evidence on the approach to treating burnout, however one study published in JAMA suggested that participation in a mindful communication program was associated with improvements in well-being, including burnout.

References:

• http://www.geripal.org/2011/04/doctor-develops-cure-for-burnout.html • Krasner MS et al. Among Primary Care Physicians Communication With Burnout,

Empathy, and Attitudes Association of an Educational Program in Mindful. JAMA. 2009;302(12):1284-1293

• http://blog.vcu.edu/dpgray/JAMA%20self-care%20end%20of%20life.pdf • Kearney MK et al. Self-care of Physicians Care for Patients at the End of Life: “Being

Connected...A Key to My Survival.” JAMA. 2009;301(11):1155-1164

Page 34: Pallimed/GeriPal Blogs to Boards - Hospice/Palliative Medicine Board Review 2012 (Q&A + Discussion)

From  Blogs  to  Boards:  AAHPM  2012  Pre-­‐Course  from  the  contributors  of  Pallimed  &  Geripal  

Page 34 of 48  

Notes Question HPM28

Your palliative care clinic team meets Nancy Bush a 46 year-old with newly diagnosed triple-negative metastatic breast cancer. She has 7 and 11 year old children. The children know Nancy has been ‘to the doctor’ a lot lately but nothing else. She is thinking about talking with the children and letting them know her diagnosis, but her mother thinks that telling them now will be too hard on them. You advise:

a) It is best to wait until Nancy’s disease is obvious to the children so their interactions with their mother will not change.

b) Telling the children now will make them too anxious. c) She should tell the older child, but the younger child is not at an appropriate

development age that he will benefit from hearing his mother has cancer. d) Telling the children of the disease may make them less anxious

Discussion: Correct answer is (d) For the purposes of the boards – your default position should be one of truthful disclosure to children of all ages. The highest quality longitudinal study of bereaved children showed:

• The early loss of a parent was associated with poverty, - contributing factors may include the loss of income, as well as the burden of medical expenses

• the increase in separation anxiety symptoms begins prior to death: this speaks to the need for preventive interventions when a family death is impending

• Several studies have shown a higher incidence of substance abuse in bereaved children, up to at least 21 months after the death

• Bereavement pattern associated with early loss of a parent was associated with poverty, substance abuse problems, and greater functional impairments.

References

• http://cases.pallimed.org/2009/05/what-do-i-say-to-my-kids.html • Rosenheim, E., Reicher, R. (1985). Informing children about a parent’s terminal

illness. J Child Psychol Psychiatry Allied Disc. 26:995-998. • Siegel, K., Raveis, V., Karus, D. (1996). Pattern of communication with children when

a parent has cancer. In L. Baider & L. Cooper (Eds) Cancer and the family, pp 109-128. John Wiley and Sons: New York.

• Psychiatric symptoms in bereaved versus nonbereaved youth and young adults: a longitudinal epidemiological study. Kaplow JB. Saunders J. Angold A. Costello EJ.

• Journal of the American Academy of Child & Adolescent Psychiatry. 49(11):1145-54, 2010 Nov.

• Sanchez L, Fristad M, Weller RA, Weller EB, Moye J. Anxiety inacutely bereaved prepubertal children. Ann Clin Psychiatry.1994;6:39-43.

• Swadi H. A longitudinal perspective on adolescent substance abuse. Eur Child Adolesc Psychiatry. 1992;1:156-170.

Page 35: Pallimed/GeriPal Blogs to Boards - Hospice/Palliative Medicine Board Review 2012 (Q&A + Discussion)

From  Blogs  to  Boards:  AAHPM  2012  Pre-­‐Course  from  the  contributors  of  Pallimed  &  Geripal  

Page 35 of 48  

Notes Question HPM29 You receive a call from the hospice nurse about a new hospice patient, Mrs. Gardner, who had a large

ischemic MCA stroke 4 months ago. She has not been able to eat, is unable to turn herself, and has developed a large stage IV decubiti on her low back. The wound measures 10cm x 8cm and 1.2cm deep. It has some limited undermining and no tunneling. At the wound bed, the spine is visible. The bed of the wound reveals malodorous, necrotic purplish muscle and tissue with extensive serosanguinous drainage. The surrounding skin is intact. Mr Gardner covers her wound with a cream but notes ‘It just keeps getting deeper.” The patient is turned q2 hours. The goal of care is to keep her comfortable and at home – a promise he made to her. The hospice nurse asks you for orders to help manage the wound. She will order an air-mattress. After washing the bed of the wound with normal saline, applying a thin layer of metronidazole gel to the base of the wound, what do you recommend for a wound care dressing?

a) Pack wound with wet-to-dry dressing and cover with ABD pad every 3 days. b) Pack wound with calcium alginate wafer and rope, cover with ABD pad every 3 days. c) Pack wound with hydrocolloid dressing and cover with ABD pad every 3 days

Discussion: (b) is correct – non-occlusive, good for wet wounds

References:

• Ferris, F., & Pirrello, R. (n.d.). Palliative Wound Care. Retrieved February 25, 2012, from google.docs presentation

• McDonald, A., & Lesage, P. (2006). Palliative management of pressure ulcers and malignant wounds in patients with advanced illness. Journal of palliative medicine, 9(2), 285-95. Mary Ann Liebert, Inc. 2 Madison Avenue Larchmont, NY 10538 USA.

• May  macerate  surrounding  skin  

Foam  Dressing    ++++  

• Can  be  used  in  infected  wounds  

Alginate  dressing  +++  

Hydrogel  ++  

• occlusive,  should  not  use  with  venous/vascular  compromise  

Hydrocolloid  +  

Transparent  Lilm  

• Can  cause  debridement  of  healing  tissue  Gauze  (wet  to  dry)  

Non-­‐adherent  dressings  

+  =  degree  of  absorption  

Page 36: Pallimed/GeriPal Blogs to Boards - Hospice/Palliative Medicine Board Review 2012 (Q&A + Discussion)

From  Blogs  to  Boards:  AAHPM  2012  Pre-­‐Course  from  the  contributors  of  Pallimed  &  Geripal  

Page 36 of 48  

Notes Question HPM30 A 45 year old man with HIV-AIDS comes to your clinic for follow-up for HIV-related

neuropathy pain. He has long declined any antiretroviral therapy, and has consistently stated he wants supportive-only care focused on maintaining his quality of life. He has a CD4 count of 90 cells/mm3. 1 year ago it was 100. He reports worsening pain control which he relates to inability to swallow his morphine ER tabs (100 mg tid) much of the time. He reports mid-throat pain, and frequently chokes on the pills, ‘gags’ them back up. Examination reveals a thin man. Mouth demonstrates scattered white plaques on the palate which reveal a red base when scraped away. Best next step is to:

a) Prescribe Nystatin ‘swish & swallow’; change morphine to 30mg elixir q4h scheduled. b) Prescribe fluconazole; change MorphineER pills to to MorphineER ‘granules’ in

pudding (such as ‘Kadian’ or ‘Avinza’ morphine formulations). c) Prescribe fluconazole, change his morphine to methadone elixir, and recommend

hospice care given his goals of care and prognosis. d) Prescribe Nystatin ‘swish & swallow’; change his morphineER to a fentanyl patch.

Discussion: Correct answer is (b) This man has HIV, AIDS, thrush, and based on the history of dysphagia and odynophagia, esophageal candidiasis as well. Due to this, systemic antifungal agents are indicated such as fluconazole. Topical agents are ineffective for esophageal candidiasis; for thrush alone they are less effective but still used as first-line agents. All of the strategies to manage his pain while he is having pill dysphagia are within the realm of reason – use of scheduled immediate release morphine, Morphine ER granules which can be given in pudding or down a G-tube (‘Kadian’ or ‘Avinza’), or transdermal fentanyl. Methadone elixir or crushed pills is a possibility, however it’s a more complicated rotation, and interacts with fluconazole, and probably not as elegant as the other solutions. Hospice care is not appropriate for the patient based on prognosis. A CD4 count of 90 which is slowly declining, and no other major life-limiting complication of HIV, indicate his expected prognosis is well over 6 months. Indeed, if he chose to start antiretrovirals it could be decades. Hospice eligibility guidelines, while not very evidence-based, suggest a CD4 count <25cells/mm3 or a persistent viral load >100,000 copies/ml, as well as a serious HIV related comorbidity such as CNS lympthoma, MAC bacteremia untreated or unresponsive to treatment, Progressive multifocal leukoencephalopathy, systemic lympthoma, visceral Kaposi’s sarcoma, renal failure, cryptosporidium infection, or toxoplasmosis unresponsive to therapy. References:

• http://www.eperc.mcw.edu/EPERC/FastFactsIndex/ff_213.htm • http://www.pallimed.org/2006/01/prognosis-in-end-stage-hiv-aids.html • http://www.eperc.mcw.edu/EPERC/FastFactsIndex/ff_147.htm

Page 37: Pallimed/GeriPal Blogs to Boards - Hospice/Palliative Medicine Board Review 2012 (Q&A + Discussion)

From  Blogs  to  Boards:  AAHPM  2012  Pre-­‐Course  from  the  contributors  of  Pallimed  &  Geripal  

Page 37 of 48  

Notes Question HPM31 Ms. F is a 64 year old who you see in your palliative care clinic 2 months after the death of her husband in

an ICU. She describes sadness over the loss of her husband, as well as waves of yearning, helplessness, and guilt over her decision to proceed with the terminal extubation of her husband. She reports sleep and appetite changes, as well as fatigue. Which of the following in the most likely diagnosis?

a) Grief b) Complicated grief disorder c) Post-Traumatic Stress Disorder d) Major depressive disorder

Discussion: Correct answer is (a) a) Ms. F’s symptoms are most likely the result of a normal response to loss –grief. Normal reactions

often come in waves and may include denial, anger, disbelief, yearning, anxiety, sadness, helplessness, guilt, sleep and appetite changes, fatigue, and social withdrawal. The frequency of feeling negative symptoms of disbelief, yearning, anger, and depressed mood has been shown to generally peak by 6 months.

b) For 10-20% of bereaved individuals, grief can become complicated and significantly impact their ability to function. It is important for clinicians to recognize and treat complicated grief (also known as prolonged or pathologic grief disorder) as it may lead to psychiatric morbidity, suicidal ideation, functional disability, and low quality of life. The symptoms of complicated, or prolonged, grief are distinct from normal grief, bereavement related depression, and anxiety disorders. Key features include separation distress with intense yearning and longing for the deceased, as well as dysfunctional thoughts, feelings, or behaviors related to the loss. Several psychotherapy treatments have been shown to be beneficial including Cognitive Behavioral therapy and Complicated Grief Treatment (CGT).

c) Even though family members of patients in the ICU are at risk for PTSD, she does not describe symptoms of 1) re-experiencing the traumatic event, 2) avoidance of situations associated with the death; and 3) increased arousal. In addition she does not describe significant social or occupational impairment.

d) Ms. L does not meet DSM-IV criteria for depression. To meet DSM-IV criteria, individuals must have at least two weeks of depressed mood or loss of interest accompanied by at least four additional symptoms of depression that include sleep disturbances, guilt and feelings of worthlessness, lack of energy, loss of concentration and difficulty making decisions, anorexia or weight loss, psychomotor agitation or retardation, and suicidal ideation

References: • http://www.geripal.org/2011/09/study-of-dignity-therapy-on-distress.html • Shear K, Frank E, Houck PR, Reynolds CF, 3rd. Treatment of complicated grief: a randomized

controlled trial. JAMA. Jun 1 2005;293(21):2601-2608. • Prigerson HG, Horowitz MJ, Jacobs SC, et al. Prolonged grief disorder: Psychometric validation

of criteria proposed for DSM-V and ICD-11. PLoS Med. Aug 2009;6(8):e1000121. • Zisook S, Simon NM, Reynolds CF, 3rd, et al. Bereavement, complicated grief, and DSM, part 2:

complicated grief. J Clin Psychiatry. Aug 2010;71(8):1097-1098. • Boelen PA, de Keijser J, van den Hout MA, van den Bout J. Treatment of complicated grief: a

comparison between cognitive-behavioral therapy and supportive counseling. J Consult Clin Psychol. Apr 2007;75(2):277-284.

• Anderson WG, Arnold RM, Angus DC, Bryce CL. Posttraumatic stress and complicated grief in family members of patients in the intensive care unit. J Gen Intern Med. Nov 2008;23(11):1871-1876

Page 38: Pallimed/GeriPal Blogs to Boards - Hospice/Palliative Medicine Board Review 2012 (Q&A + Discussion)

From  Blogs  to  Boards:  AAHPM  2012  Pre-­‐Course  from  the  contributors  of  Pallimed  &  Geripal  

Page 38 of 48  

Notes Question HPM32 You are awoken at 2am the day before your interdisciplinary hospice team meeting, by a nurse who

just joined your hospice. She is visiting Mr. Gunter Liszt, an 89 year old who is on hospice with end-stage heart disease. He has an ejection fraction of 21%, is oxygen dependent. He was last hospitalized for recurrent pulmonary edema and systolic blood pressure in the 80s. During his hospitalization, he developed cardiorenal syndrome and has become oliguric, with a daily urine output of 80-100mls. He is on Amiodarone 600mg daily, furosemide 40mg bid via PICC line, metoprolol 25mg bid, oxycodone 5mg prn dyspnea, lorazepam 0.5mg SL prn. He had been weaned off of his dobutamine drip a day prior. He was discharged back to his nursing home with hospice services 4 days ago. During that time, he became increasingly somnolent, with peripheral mottling, and over the last 2 days has become anuric. His family was called by the nurse, and was notified that his prognosis was poor: likely hours to days. He is DNR/DNI and you learn that has an implanted pacemaker and defibrillator. The family is at the bedside distraught, watching him, the nurse nervously explains. “He is actively dying, but every few minutes he jumps out of bed –the defibrillator is shocking him again and again. His heart rate is erratic and in the 150s. He is moaning from time to time. What can I do?” What is the next best thing to do?

a) Comfort the family while waiting for the defibrillator company to send a representative out to deactivate the defibrillator.

b) Restart the dobutamine drip. c) Start a morphine drip at 1mg/hour. d) Tape a magnet to his chest over the defibrillator. e) Give the patient lorazepam 1mg IV push now. f) Review with the admissions team the importance of identifying patients with AICDs, and

having associated goals of care about deactivation Discussion: Correct answer is (d)

a) Calling the defibrillator company is a good step, but only after placing a magnet over his chest. Waiting for a representative to come may take too long. Additionally, hopefully the admissions nurse already knows which company the AICD if from or which cardiologist placed it. This should be documented in the admission package.

b) Restarting the dobutamine drip will not reverse the process, probably will aggravate any arrhythmia, and plays no role.

c) Morphine is contraindicated in renal failure, and while giving him analgesics is indicated, stopping the source of his suffering is a more urgent priority

d) Taping a magnet to the chest is the best initial step – followed by calling the company to have a representative come to turn off the device.

e) Lorazepam will not change the circumstance, and the patient is already mostly unresponsive.

f) While this is important, it is not the first step. As per recommendations from NHPCO all hospices should have policies to address AICD management in their patients. All patients with AICDs should be identified, the companies known. Discussions about deactivating the AICD should be held early in the admission process.

References: http://www.nhpco.org/files/public/NHPCO_ICD_position_statement_May08.pdf http://www.pallimed.org/2008/06/deactivating-icd-guidelines.html http://www.pallimed.org/2010/03/implantable-cardiac-defibrillators.html

Beets, M. T., & Forringer, E. (2011). Urgent implantable cardioverter defibrillator deactivation by unconventional means. Journal of pain and symptom management, 42(6), 941-5. doi:10.1016/j.jpainsymman.2011.02.025

Page 39: Pallimed/GeriPal Blogs to Boards - Hospice/Palliative Medicine Board Review 2012 (Q&A + Discussion)

From  Blogs  to  Boards:  AAHPM  2012  Pre-­‐Course  from  the  contributors  of  Pallimed  &  Geripal  

Page 39 of 48  

Notes Question HPM33, 34

A 40 year old man is at home receiving hospice care for metastatic bladder cancer to peritoneum, lung, pleura, and spine. His ECOG is 4. His back and chest wall pain had been well controlled on morphineCR 100mg bid, dexamethasone 4mg bid.

In the last week he has become progressive more dyspneic, such that he reports severe air hunger at rest, despite medication changes including dexamethasone 8mg bid, lorazepam 2 mg q6h, and transitioning to a morphine PCA which is now at 8mg/hour with a 8mg/10 minute PCA dose. He is normoxic on 4lpm of O2 by nasal cannula. He declines inpatient admission for evaluation and symptom relief. You think he has less than a week to live.

He tells you, “I can’t go on like this – this is not how I wanted to die. Can’t you put me sleep so I don’t have go through this?”

After evaluation by the hospice interdisciplinary team, phone consultation with a colleague, and discussions with the patient and his family, you decide to sedate the patient to unconsciousness, with no plan of lightening the sedation HPM33: The best term to describe what the patient is asking for is:

a) Physician assisted suicide b) Proportionate palliative sedation c) Terminal sedation d) Deep, continuous sedation

HPM34: His medication treatment plan should include which of the following:

a) Escalation of his morphine continuous rate until he is unarousable b) Anticholinergic medication to minimize retained oropharyngeal secretions c) Intravenous normal saline to prevent dehydration d) Bolus and continuous intravenous midazolam or pentobarbital to maintain a state of

unresponsiveness Discussion: answers: HPM33: (d) – HPM34: (b and d)

a) Physician Assisted Suicide (PAS) describes the practice of a terminally ill patient self-administering a lethal dose of a medication, prescribed by a physician for that purpose.

b) Proportionate palliative sedation describes a wide spectrum of sedating practices in which patients are given sedating drugs in order to relieve a symptom, but the fundamental goal is to alleviate the underlying symptom with as little sedation as necessary.

c) The practice known as terminal sedation is now preferably called (some variation of) ‘continuous deep sedation’ (CDS) or ‘palliative sedation to unconsciousness’, and implies a decision to deliberately, pharmacologically induce a coma/state of unresponsiveness, usually with the intent to maintain that state until death. As practiced, life-prolonging treatments are usually withheld, although nothing absolutely mandates this. It is widely accepted as an ethically sound practice for dying patients, close to death, with refractory suffering. It is controversial outside of that situation (for patients with longer prognoses for instance).

d) CDS is best done by using bolus (especially to initiate the coma) and then continous infusions of a sedating medication such as a barbiturate, benzodiazepine, or propofol. It should not be done with opioids as they are inadequate as sedatives, and escalating doses can cause myoclonus and seizures. Routine end of life cares should be given including secretion management. IV hydration has no coherent indication in these situations in which life is no longer being prolonged and a patient is comatose.

References:

• http://www.pallimed.org/2009/09/palliative-sedation-in-annals.html

Page 40: Pallimed/GeriPal Blogs to Boards - Hospice/Palliative Medicine Board Review 2012 (Q&A + Discussion)

From  Blogs  to  Boards:  AAHPM  2012  Pre-­‐Course  from  the  contributors  of  Pallimed  &  Geripal  

Page 40 of 48  

Notes Question HPM35 Mr. Xiao is a 63-year-old edentulous Chinese immigrant, long-standing smoker who is on hospice for

an unresectable fungating squamous cell carcinoma at the base of his tongue with metastasis to lung. He is getting nutrition and medications through a feeding tube. His medications currently include: Methadone 70mg tid, oxycodone 60mg q2 hours prn, dexamethasone 8mg daily, scopolamine patch 1.5mg, topical viscous lidocaine mixed with thrombin powder swish and spit prn. He and his wife come in distraught due to his pain, bleeding, and most of all the malodor. He also often gags on his own blood. He hopes to have his grandchildren visit soon, and does not want them to be put off by smell in his mouth. He tells you of a friend of his from the infusion suite who received a single-fraction of palliative radiation for his painful metastasis to his femur and asks if there is a similar approach for him. Due to the anatomy of the tumor, palliative embolization is not an option. Which of the following treatment options is the most appropriate?

a) Single-fraction radiation b) Hypo-fractionated Radiation therapy c) Standard radiation therapy

Discussion: Correct answer is (b)

a) While there is increasing literature to support the use of hypofractionated or single fraction radiation to painful bone metastases, there are some conditions where single fraction radiation is still contraindicated. Radiation to a lesion at the base of the tongue is one such setting. Despite this, literature is showing that hypofractionated radiation therapy (5-14 fractions) may be helpful to palliate severe symptoms in this setting. Anytime one irradiates the mouth, symptom management of side effects must be considered.

b) Irradiation of the mouth can lead to severe mucositis, dry mouth, dental disease, and thrush. In patients with teeth, this is particularly concerning. Patients with fillings need to have dental guards to prevent local ulceration. Even patients without teeth need to have close symptom management for acute and subacute symptoms from the radiation therapy.

c) Using mouthwashes such as Biotene® or a simple combination of baking-soda and salt in water may help the severe xerostomia. Mucositis and thrush need to be treated throughout the course of radiotherapy and for a week to 10 days post-therapy, or as long as symptoms are present.

References:

• http://www.pallimed.org/2007/04/proxies-single-fraction-radiotherapy.html • http://jco.ascopubs.org/content/25/11/1423.abstract Chao E et al. Palliative Radiotherapy

Trials for Bone Metastases: A Systematic Review. JCO 2007;25(11) 1423-36. • http://www.ingentaconnect.com/content/ftd/era/2010/00000010/00000003/art00009 • McLarnon C et al. Quality-of-life considerations in treatment of unresectable, recurrent head

and neck cancer. 2010;10(3) 345-352. • Porceddu, S. V., Rosser, B., Burmeister, B. H., Jones, M., Hickey, B., Baumann, K., Gogna,

K., et al. (2007). Hypofractionated radiotherapy for the palliation of advanced head and neck cancer in patients unsuitable for curative treatment--“Hypo Trial”. Radiotherapy and oncology  : journal of the European Society for Therapeutic Radiology and Oncology, 85(3), 456-62. Retrieved from http://dx.doi.org/10.1016/j.radonc.2007.10.020

Page 41: Pallimed/GeriPal Blogs to Boards - Hospice/Palliative Medicine Board Review 2012 (Q&A + Discussion)

From  Blogs  to  Boards:  AAHPM  2012  Pre-­‐Course  from  the  contributors  of  Pallimed  &  Geripal  

Page 41 of 48  

Notes Question HPM36 You are a medical director for a hospice agency.

MF, an 83 yo woman with metastatic breast cancer leading to bone marrow infiltration and chronic cytopenias has been receiving hospice care at home with your agency for 2 months. She is ambulatory, and spends half her time in bed or a chair due to weakness and fatigue, but rates her quality of life as very high. She has confirmed with her RN case manager her goals are to have as good a quality of life as possible, but is not interested in further treatments to prolong her life. At a follow-up visit with her oncologist who is her hospice attending physician, MF complains of worsening and distressing fatigue. The oncologist orders a CBC, which showed a hemoglobin of 7.4mg/dL and a hematocrit of 23. 3 months ago they were 9.5mg/dL and 29. She orders a transfusion of 2U PRBC; the patient indicates interest in receiving it. You call the oncologist to discuss the transfusion and she says tells you it has helped the patient’s fatigue in the past and hopes it will help her now. The best next step would be to:

a) Decline to cover the costs of the transfusion as it is unrelated to the patient’s ‘terminal’ diagnosis

b) Agree that the hospice will cover the costs of the transfusion, and will monitor her for signs of improvement

c) Recommend that the patient receive oral iron supplementation and methylphenidate d) Discharge the patient from your agency’s care, as she has chosen to seek life-

prolonging treatments for her cancer. Discussion: Correct answer is (b)

a) Anemia is clearly related to her cancer diagnosis, and the hospice agency has assumed responsibility for all palliative treatments related to that diagnosis.

b) There are no data to guide us about when/who to transfuse for symptomatic anemia in hospice and EOL settings. Anecdote, and common sense, indicate that some people have subjective improvement in energy/fatigue with transfusions; some clearly do not. This has been an effective tx for the patient in the past, and she is ambulatory. In addition, the hospice agency runs the risk of sending a message to the patient and her doctor that patients have to forego expensive treatments, even if palliative, and may undermine the patient’s relationship with her oncologist

c) Iron supplementation is unlikely to help with the patient’s anemia given its cause (marrow infiltration). There is no known role for psychostimulants in anemia-associated fatigue.

d) Despite the question of benefit of transfusion for fatigue, there is even less reason to think it will actually prolong her life. In this situation, the intent and likely outcome of transfusion is strictly palliative.

References:

• http://www.pallimed.org/2009/03/hospice-patients-feel-abandoned-by.html

Page 42: Pallimed/GeriPal Blogs to Boards - Hospice/Palliative Medicine Board Review 2012 (Q&A + Discussion)

From  Blogs  to  Boards:  AAHPM  2012  Pre-­‐Course  from  the  contributors  of  Pallimed  &  Geripal  

Page 42 of 48  

Notes Question HPM37 JW is a 48 year old female with metastatic osteosarcoma of her L pelvis (mets to lung and

spine), undergoing chemotherapy. She is hospitalized for neutropenic fever; her hospital course has been unremarkable. Her albumin is 2.7g/dl, calcium is 10.4mg/dl, and LDH 700IU/L. She has a poor appetite and has lost 5 kilos in the last 2 months. She has been getting steadily weaker and now needs assistance with housework, and occasionally needs help getting up from a chair/toilet. She understands her cancer is incurable, and asks you how much time you think she has to live. She tells you her oncologist told her ‘It was in God’s hands.’ The best initial response is:

a) I am God: 63 days. b) I can’t understand why all these oncologists never tell their patients the truth! c) Most likely 1-3 months d) Most likely 4-6 months

Discussion: (c) is the correct answer

• Impersonating God is usually a bad idea, as is throwing your colleagues under the bus. She has an incurable, metastatic tumor, hypoalbuminemia, elevated LDH, weight loss, progressive functional decline, and a PPS/KPS of 60. In general, performance status has been considered to be the single most powerful prognostic factor in cancer, with rule-of-thumb guidelines for patients with solid tumors and declining PS that KPS 40-60 have median survivals in the 1-3 mo range. Less than 40 median survival is less than a month. Other factors independently help predict survival as well including presence of dyspnea, hypoalbuminemia, elevated LDH, hypercalcemia (median survival 6-8wk), leptomeningeal disease (median survival 21 days). A recent study of hospitalized cancer patients used a combined score of KPS, number of metastatic sites, Albumin level, and LDH to predict 2 month survival. In that model, her median survival is ~50 days.

• Prognostic disclosures, in patients who want to hear them, are best done by giving accurate estimates with ranges, as well as disclosure of uncertainty. There is ongoing work on whether using graphs or graphic representations, and whether disclosures are best are as survival statistics over time (eg ‘80% survival at 6 months, meaning 4 out of every 5 patients with your condition would be alive at 6 months’), vs estimates of survival length, or combinations of these. There is no clear professional standard about which one is best. Using at least one rhetorically negative phrase helps patients retain negative information better (e.g. ‘The results were not good…’). Patients, in research settings, say they want as much info in as many ways as possible. For the boards, probably frank & honest is the best policy.

References:

• http://www.pallimed.org/2008/06/2-month-prognosis-in-hospitalized.html • http://www.pallimed.org/2009/01/prognosis-of-leptomeningeal.html • http://www.pallimed.org/2007/08/hypercalemia-of-malignancy-palliative.html

Page 43: Pallimed/GeriPal Blogs to Boards - Hospice/Palliative Medicine Board Review 2012 (Q&A + Discussion)

From  Blogs  to  Boards:  AAHPM  2012  Pre-­‐Course  from  the  contributors  of  Pallimed  &  Geripal  

Page 43 of 48  

Notes Question HPM38 Ms. L is a 46 year old who is currently receiving treatment for metastatic breast cancer. She

has 3 children ages 8-13. Ms. L comes to your office complaining of fatigue. She states that she wants to participate more with her children’s lives but symptoms of fatigue limit what she can do. She denies difficulty initiating or maintaining sleep. At times she is tearful but there are activities, such as watching movies with her children, that bring her joy. She denies worthlessness or excessive guilt. Which of the following has the best evidence to improve her symptoms?

a) Structured Exercise program b) Paroxetine c) Limit energy expenditures d) Megestrol

Discussion: (A) is the correct answer

a) Trials of moderate exercise have demonstrated significant benefits in patients with cancer. Improvements have been shown to include less fatigue, decreased sleep disturbance, improved functional capacity, and better quality of life.

b) Typical antidepressants such as SSRIs, including paroxetine, have not shown to be beneficial for fatigue outside of patients who are depressed. Ms. L’s current symptoms do not point to a diagnosis of depression.

c) There is some evidence for a modest benefit from energy conservation and activity management (ECAM) although only limiting energy expenditures as answered in C would be insufficient to address Ms. L’s concerns. Decreasing activity may lead to further muscle wasting, loss of physical strength, and worsening endurance.

d) There is no evidence that megestrol improves cancer related fatigue References:

• Segal R, Evans W, Johnson D, et al. Structured exercise improves physical functioning in women with stages I and II breast cancer: results of a randomized controlled trial. J Clin Oncol. 2001;19(3):657

• Oldervoll LM, Loge JH, Paltiel H et al. The effect of a physical exercise program in palliative care: A phase II study J Pain Symptom Manage. 2006 May;31(5):421-30.

• Morrow GR, Hickok JT, Roscoe JA, et al. Differential effects of paroxetine on fatigue and depression: a randomized, double-blind trial from the University of Rochester Cancer Center Community Clinical Oncology Program. J Clin Oncol. 2003;21(24):4635.

• Barsevick AM, Dudley W, Beck S, et al. A randomized clinical trial of energy conservation for patients with cancer-related fatigue. Cancer. 2004;100(6):1302

Page 44: Pallimed/GeriPal Blogs to Boards - Hospice/Palliative Medicine Board Review 2012 (Q&A + Discussion)

From  Blogs  to  Boards:  AAHPM  2012  Pre-­‐Course  from  the  contributors  of  Pallimed  &  Geripal  

Page 44 of 48  

Notes Question HPM39 Mr A is a 67 year-old Spanish-speaking man with metastatic pancreatic cancer diagnosed

during this hospital stay. The physician on the palliative care team, Dr. S is organizing a family meeting to discuss prognosis and goals of care. The meeting is to include Mr. A and his son, the oncology team, and the social worker on the palliative care team. Mr A speaks only Spanish. His son speaks both English and Spanish, as does Dr. S. The son says that he has been been acting as the interpreter during previous meetings with physicians. The best next step would be:

a) Use a professional interpreter, but Dr. S should avoid short phrases in English as they often don’t give enough context to give an accurate interpretation

b) Mr. A’ son should act as the interpreter if it is ok with Mr A c) Dr. S should conduct the family meeting in Spanish d) Use a professional interpreter and ensure that the he/she is briefed before the family

meeting

Discussion: (D) is the correct answer

a) Simple language and short phrases are easier to interpret accurately. b) The son may not accurately translate what is said by the physician. This may be due to

a lack of familiarity with medical terminology in either language, discomfort with what is being discussed in the family meeting, or overwhelming emotions or personal beliefs.

c) Physicians who can speak a second language should use an interpreter unless they have native fluency in that second language and know medical terminology in that second language. Another reason that Dr S should consider not running the family meeting in Spanish is that other health care providers at the family meeting will still require interpretation services.

d) Medical interpreters have expertise in interpretation as opposed to ad hoc interpreters (i.e. family members, hospital staff). Errors in interpretation occurs less commonly with professional interpreters and patient satisfaction is higher. Professional interpreters should be briefed before a family meeting especially if a difficult conversation will take place including delivering bad news or discuss end-of-life issues.

References: • Schenker Y, et al. ‘‘Her Husband Doesn’t Speak Much English’’: Conducting a

Family Meeting with an Interpreter. J Palliat Med. 2011 Nov 22. (Epub) • Butow PN, Goldstein D, Bell ML, et al. Interpretation in consultations with immigrant

patients with cancer: how accurate is it? J Clin Oncol. 2011;29(20):2801-2807

Page 45: Pallimed/GeriPal Blogs to Boards - Hospice/Palliative Medicine Board Review 2012 (Q&A + Discussion)

From  Blogs  to  Boards:  AAHPM  2012  Pre-­‐Course  from  the  contributors  of  Pallimed  &  Geripal  

Page 45 of 48  

Notes Question HPM40 You are called to see a 13 year old for a palliative care consultation. You call the pediatric

oncologist who tells you the boy has had sarcoma for 4 years, and since he has been hospitalized for the latest round of cancer treatment the pain has become much worse. The oncologist shares that recent scans show the latest treatment is not working and there are no further measures he would offer, but he is not sure if he will discuss this with the patient himself. The parents know, and have been asking for increased pain medication, but the attending is concerned they are trying to “you know…move things along.” The best initial response to his concern is

a) “It is a myth that opioids hasten death at the end of life” b) “They probably are trying to protect him from finding out what is going on” c) “Thoughts of hastening a child’s death are not uncommon in these circumstances, but

it’s usually because of uncontrolled pain.” d) “If you tell the patient the truth, the parent’s grief will be diminished.”

Discussion: Correct answer is (c)

a) While that is generally true (morphine myth), it is pedantic and off-topic, and bad consultation form.

b) B is psychological speculation c) C is true. In one study, 36% (49 of 136, about 1 in every 3) of parents of deceased

children, in retrospect, would have considered discussing hastening death under certain circumstances with a breakdown of scenarios given. Uncontrollable pain was the most common circumstance to elicit a hypothetical consideration of HD. 15% would have considered HD for non-physical suffering. Bottom line: thoughts of hastening death are common. This phrase, followed by an offer to explore the situation with the family further, is the best response to the requesting physician.

d) D is tricky. Research supports that adolescents want disclosure of information to them, and part of this consultation should involve addressing the question of what the patient himself should be disclosed. For the boards, of course the default posture should be for one of disclosure to children. However this response is premature, and not addressing the reason you are being consulted.

References: • http://www.pallimed.org/2009/12/attitudes-of-adolescent-cancer.html. • Dussel V, Joffe S, Hilden JM, Watterson-Schaeffer J, Weeks JC, & Wolfe J.

(2010)Considerations about hastening death among parents of children who die of cancer.Archives of pediatrics & adolescent medicine, 164(3), 231-7. PMID: 20194255

• Heath JA, Clarke NE, Donath SM, McCarthy M, Anderson VA, & Wolfe J. (2010) Symptoms and suffering at the end of life in children with cancer: an Australian perspective. The Medical journal of Australia, 192(2), 71-5. PMID: 20078405

• When Children Die: Improving Palliative and End-of-Life Care for Children and Their Families. Institute of Medicine 2002.)

• Pousset G, Bilsen J, De Wilde J, Benoit Y, Verlooy J, Bomans A, Deliens L, & Mortier F. (2009) Attitudes of adolescent cancer survivors toward end-of-life decisions for minors.Pediatrics, 124(6). PMID: 19948616

Page 46: Pallimed/GeriPal Blogs to Boards - Hospice/Palliative Medicine Board Review 2012 (Q&A + Discussion)

From  Blogs  to  Boards:  AAHPM  2012  Pre-­‐Course  from  the  contributors  of  Pallimed  &  Geripal  

Page 46 of 48  

Notes Question HPM41 Your hospital ethics committee asks you to weigh in on a challenging case in the emergency

room: a 3 month old baby, Marisol, with anencephaly has been brought to your publically-funded hospital from a private hospital. The baby is clearly having difficulty breathing and the mother (who has sole custody) is requesting that her baby be intubated for ventilatory support. "They wouldn't do it at Hospital Private, but told me you had to. After all, only God should determine when a baby dies - not you, nor I." The pediatrician and neurologist both reinforce that with or without a ventilator, "it is a matter of time" before the baby dies. "Intubation seems to us medically inappropriate,” they state. “We will do the right thing, morally and legally. We will take your advice.” What is your hospital's legal obligation in this case?

a) Baby Marisol must be intubated, and should move to establish a court-appointed guardian.

b) Baby Marisol should not be intubated, instead the focus should be on comfort at end-of-life.

c) Baby Marisol should not be intubated, and move to establish a court-appointed guardian.

Discussion: (a) is the correct answer Baby Doe Rules were implemented in the 1980s after the case of Baby Doe, an infant born with a tracheo-esophageal fistula and Downs whose parents opted not to consent for surgery after hearing of the low chance of survival and the life-long disabilities. The hospital staff requested a guardian to overturn the parents’ decision, but the Indiana Court ruled in favor of the parents. US Department of Health and Human Services (HHS) responded by issuing a ruling that babies born with disabilities in publically funded hospitals must not be discriminated against. Conflict between the right-to-life groups and medical communities resulted in compromise with Baby Doe Regulations in 1985: where all babies must receive appropriate nutrition, medications, and hydration – with 3 exceptions: infant is chronically/irreversibly comatose, the treatment would merely prolong dying, and the treatment would not improve survival and in turn be considered inhumane. This continues to cause much concern – ethicists express the concern that physicians will respond by providing aggressive overtreatment. However, the ruling also supports the definition of appropriate as derived from a physician’s clinical judgment. A concern is that quality of life is seen as a subjective and not clinical position according to the Baby Doe Regulations. Numerous legal cases followed that of Baby Doe, including noted Baby K in Washington DC. Baby K was born. The court decision based their decision on EMTALA regulation - a law the obligates publically-funded hospitals to treat patients who come to the emergency room. The judge, while recognizing the moral distress of the physicians, stated that allowing the hospital to not intubate baby K would open Pandora's box and potentially establish a precedent to allow hospitals to turn away patients in respiratory distress from motor vehicle accidents if they had an underlying terminal diagnosis, "on the grounds that they would die anyway." Furthermore, the CAPTA (Child Abuse Protection and Treatment Act) rulings, remove all quality-of-life from the decision-making calculation regarding whether a treatment is medically indicated or not under the Baby Doe regulations. References:

• Pope, T. (2009). Medical Futility Blog: Georgia State University Law Review, “Baby Doe at Twenty-Five.” Medical Futility Blog. Retrieved March 5, 2012, from http://medicalfutility.blogspot.com/2009/12/georgia-state-university-law-review.html

• Morrow, J. (2000). Making Mortal Decisions at the Beginning of Life: The Case of Impaired and Imperiled Infants. JAMA: The Journal of the American Medical Association, 284(9), 1146-1147. doi:10.1001/jama.284.9.1146

• Romesberg, T. L. (2003). Futile Care and the Neonate: In The Matter of Baby K. Advances in Neonatal Care, 3(4), 213-219. Retrieved from

Page 47: Pallimed/GeriPal Blogs to Boards - Hospice/Palliative Medicine Board Review 2012 (Q&A + Discussion)

From  Blogs  to  Boards:  AAHPM  2012  Pre-­‐Course  from  the  contributors  of  Pallimed  &  Geripal  

Page 47 of 48  

http://www.medscape.com/viewarticle/464018_4 • Pope, T. (2009). Medical Futility Blog: Georgia State University Law Review, “Baby

Doe at Twenty-Five.” Medical Futility Blog. Retrieved March 5, 2012, from http://medicalfutility.blogspot.com/2009/12/georgia-state-university-law-review.html

• Sahler OJZ, Frager G, Levetown M, Cohn FG, Lipson MH: Medical education about the end of life care in a pediatric setting: principles, challenges, and opportunities. Pediatr 2000;105:575-84.

Page 48: Pallimed/GeriPal Blogs to Boards - Hospice/Palliative Medicine Board Review 2012 (Q&A + Discussion)

From  Blogs  to  Boards:  AAHPM  2012  Pre-­‐Course  from  the  contributors  of  Pallimed  &  Geripal  

Page 48 of 48  

Contact information for blogger presenters (in alphabetical order): Suzana Makowski E: [email protected] or [email protected]

Twitter: @suzanakm Drew Rosielle E: [email protected]

Twitter: @drosielle Christian Sinclair E: [email protected]

Twitter: @ctsinclair Paul Tatum E: [email protected]

Twitter: @doctatum Eric Widera E: [email protected]

Twitter: @ewidera